Sie sind auf Seite 1von 134

Author:

mentaldental

ID:

94988

Filename:

Misc NBDE Test.txt

Updated:

2011-07-24 06:59:55

Description:

NBDE

Flashcards app available on theAndroid Marketplace as well as on the Apple App


Store for iOS.

1. Arthus Reaction
Type III Hypersensitivity
- antibody-antigen complex after intradermal injection of an antigen
- causes local vasculitis bc immune complexes deposit in dermal blood vessels
- PMNs come in and result in inflamm response

2. Which of the following drugs is likely to aid in treating or preventing influenza


in high risk subjects during epidemics?A) StilbamidineB) AcyclovirC)
AmantadineD) Amphotericin BE) Griseofulvin
Amantadine

3. Which of the following positions would yield the smallest measurement of


vertical dimension?
A) Edge-to-edge
B) Maximum intercuspation
C) Reverse overlap

D) Retruded contact
B) maximum intercuspation

4. Fordyce's granules/spots in mucous membrane of cheek


ectopic/aberrant sebaceous glands- departs from normal, but not bad

5. Moving the mandible from a maximum intercuspal position to a retruded


contact position usually results in
A) all of these.
B) increased occlusal vertical dimension.
C) increased horizontal overlap.
D) decreased vertical overlap.
A) all of these
- MIP- gives the lowest value of occlusal vertical dimension bc all cusps are
maximully intercuspated

6. Which of the following characterizes the alveolar bone proper of a tooth


socket?
A) It contains haversian systems, has collagen fibers in its matrix, and is about
96 per cent inorganic salts.
B) It serves to attach free gingival periodontal ligament fibers, and is a part of
the total tooth attachment apparatus.
C) It resorbs when subjected to pressure, and contains numerous openings
through which vessels pass.
D) It consists of compact bone, shows dark on a radiograph, and contains
Sharpey's fibers.
C)

7. A broad, flat facet existing on the outer aspect of the mesiolingual cusp of a
maxillary first molar, and running in a mesiolingual to distofacial direction, was
probably caused by which of the following contacting movements?

A) ProtrusiveB) WorkingC) Lateral protrusiveD) Non-working


B) working

8. Which of the following premolars has a mesial marginal ridge that is more
cervically located than its distal marginal ridge?
A) Mandibular first
B) Mandibular second
C) Maxillary second
D) Maxillary first
A) mandibular first
maxillary first has mesial marginal groove and mesial buccal cusp ridge is
longer than distal cusp ridge

9. During delayed hypersensitivity reactions, sensitized small lymphocytes


produce
A) leukotrienes.B) lymphokines.C) antibodies.D) none of these.E) histamine.
B) lymphokines (interleukins= cytokines)
- leukotrienes are produced by arachidonic acid

10.Collagen degradation observed in chronic periodontal disease may result


directly from the action of collagenase enzymes of oral microbial origin. Such
enzymes have been demonstrated as component systems of
A) Leptothrix buccalis.
B) Entamoeba gingivalis.
C) Streptococcus faecalis.
D) Veillonella alcalescens.
E) Bacteroides species
E) bacteroides species

11. The spacing between anterior teeth in the primary dentition is most frequently
caused by
A) tongue thrusting.

B) the growth of the dental arches.


C) the pressure from succedaneous teeth.
D) thumb-sucking.
B)

12.The first succedaneous tooth to erupt is the


A) permanent mandibular central incisor.
B) primary mandibular first molar.
C) permanent mandibular first molar.
D) primary mandibular central incisor.
E) maxillary central incisor.
A)
because succedaneous means a permanent tooth that succedes a primary tooth

13.The cervical line on adult teeth has the greatest depth of curvature toward the
incisal on the
A) facial aspect.
B) lingual aspect.
C) dentinoenamel junction.
D) mesial aspect.
E) distal aspect.
D) mesial

14.Behcet's Syndrome
o - most common in turkey and Japan
o - mouth and genital ulcers
o - inflamed cartilage
o - vasculitis

15.Which of the following enteric microorganisms is NOT typically found in


normal human intestinal flora?A) Salmonella typhi
B) Bacteroides fragilis
C) Escherichia coli
D) Pseudomonas aeruginosa
E) Proteus vulgaris
Salmonella typhi causes typhoid fever; motile

16.In a longitudinal section of a premolar crown, the enamel is thickest in the


A) occlusal third.B) cervical third.C) junction of the cervical and middle
thirds.D) middle third.
A) occlusal 1/3

17.The type of infection most commonly transmitted by transfusion of properly


screened blood is
A) type B hepatitis.
B) non-A non-B hepatitis.
C) type A hepatitis.
D) cytomegalovirus.
o B) Hepatitis C
o - Chronic, Cirrhosis, Carcinoma, Carriers

18.The connective tissue sheath that surrounds a muscle as a whole and is


synonymous with the gross anatomic deep fascia is the
A) periosteum.B) perimysium.C) perichondrium.D) endomysium.E)
epimysium.
endomysium (individual muscle fibers)--> perimysium (bundle of fibers)-->
epimysium (bundle of fascicles)

19.Salivary, sweat, sebaceous, and von Ebner's glands all have in common the
characteristic of being

A) exocrine.B) simple.C) holocrine.D) merocrine.E) compound.


exocrine glands
Exocrine glands are named apocrine gland, holocrine gland, or merocrine gland
based on how their product is secreted.
Apocrine glands - a portion of the plasma membrane buds off the cell,
containing the secretion.
Holocrine glands - the entire cell disintegrates to secrete its substance.
Merocrine glands - cells secrete their substances by exocytosis.

20.In an ideal intercuspal position, the cusp tip of a permanent maxillary canine
should contact
A) the mandibular lateral incisor only.
B) no other tooth.
C) the mandibular canine only.
D) the mandibular first premolar only.
E) both mandibular canine and first premolar.
B) No other tooth

21.Which of the following primary teeth has the smallest faciolingual dimension
of its crown?
A) Maxillary canine
B) Mandibular central incisor
C) Maxillary lateral incisor
D) Mandibular lateral incisor
B) Mandibular central incisor

22.The lingual height of contour on a permanent mandibular second molar is


located
A) at the junction of the middle and occlusal thirds.
B) in the occlusal third.

C) in the middle third.


D) in the cervical third.
E) at the junction of the cervical and middle thirds.
C) middle 1/3
All facial HOC= cervical
Linguals:
all anterior teeth= cervical
all posterior teeth= middle 1/3

23.Connective tissue proper is characterized as having


A) sensitivity as its main function.
B) little tissue fluid.
C) more intercellular material than cells.
D) poor reparative ability.
C)
collagen is main protein of CT

24.Prolonged administration of streptomycin may result in damage to which of the


following nerves?
A) FacialB) TrigeminalC) OpticD) Auditory
o Auditory- hearing loss (ototoxicity)
o - bactericidal--> protein synthesis inhibitor that binds 30S ribosomeprecents synthesis of proteins
o - aminoglycoside
o - IM injection

25.The non-working pathway of the maxillary cusps on the mandibular posterior


teeth is toward the

A) mesiolingual.B) distofacial.C) distolingual.D) mesiofacial.


B) distofacial
the mandibular nonworking cusps move mesiofacially

26.Properdin
o - aka Factor P
o - protein in blood
o - innate immune system
o - component of alternative pathway for complement activation
"properdin pathway"--> complexes w/ C3b
o - alternative "properdin" pathway is activated by IgA immune
complexes, bacterial endotoxins, cell walls, polysaccharides--> results in
anaphylatoxins, opsonins, chemotactic factors, MAC to fight pathogens

27.Proprioceptors are found in each of the following EXCEPT


A) skeletal muscle.
B) the pulp of a tooth.
C) the temporomandibular joint.
D) gingiva.
E) the periodontal ligament
B) pulp of the tooth
- pulp of tooth
Zones of pulp:
(Outside--> inside)
1. odontoblastic layer- odontoblasts lining pulp chamber
2. Cell free zone of Weill- acellular
- contains Rashkow's plexus (parietal plexus of nerves) and mesenchymal cells
3. cell rich zone= fibroblasts and undifferentiated mesenchymal cells
4. pulp core- fibroblasts, leukocytes, blood and lymph vessels, myelinated
(mostly A-delta and C fibers), collagen I and II; no elastic fibers

28.Which of the following permanent incisors most frequently have concave areas
on both mesial and distal root surfaces?
(a) Maxillary central; (b) Mandibular central; (c) Maxillary lateral; (d)
Mandibular lateral
B and D

29.In a patient with a left canine protection, the mesiolingual surface of the
maxillary right first molar contacts the distofacial surface of the mandibular
right first molar during a left lateral excursion. This contact is
A) a working side interference.
B) normal.
C) normal, and a non-working side interference.
D) evidence of group function.
E) a non-working side interference.
E) a non-working side interference
group function- when the working side canines and posterior teeth contact, but
they disarticulate teeth on the non-working side
ESSENTIAL TO HAVE NO NON-WORKING SIDE CONTACTS bc they are
damaging

30.In an ideal permanent tooth relationship, the tip of a mandibular canine in


lateral excursion passes
A) through the embrasure between the maxillary canine and first premolar.
B) directly in line with the maxillary canine cusp tip.
C) distal to the tip of the maxillary canine cusp.
D) mesial to the tip of the maxillary canine cusp.
D) mesial to the tip of the maxillary canine cusp

31.Salivary gland striated ducts are composed of which of the following types of
epithelium?

A) Simple low columnar


B) Psuedostratified ciliated columnar
C) Simple squamous
D) Simple cuboidal
E) Stratified squamous
A) Simple low columnar in striated ducts
-modify salivary fluid by secreting HCO3-and K+ and reabsorbing Na+ and Clusing the Na-K pump and the Cl-HCO3 pump
- ion pumping so it has a lot of mitochondria
- striated bc of basal cytoplasmic processes
Intercalated Duct
- cuboidal secretory cells
terminal secretory unit--> intercalated duct--> striated duct

32.It is possible to distinguish histologically between the stomach and the


duodenum because of the presence of
A) simple columnar epithelium lining the stomach only.
B) smooth muscle in the external musculature of the duodenum only.
C) submucosal glands in the duodenum only.
D) mucosal glands in the stomach only.
E) muscularis mucosa in the stomach only.
C)
Stomach:
- simple columnar
- rugae- folds in stomach that accomodate expansion
- produces chyme
4 types of cells located throughout the mucosa, but not submucosa
1.mucous cells
2. chief cells- pepsinogen
3. parietal cells- HCl and intrinsic factor
4. enteroendocrine cells (APUD) cells- gastrin

33.In a dry-heat oven, which of the following temperatures is aufficient for


achieving sterilization in 1-2 hours?
A) 121oC
B) 160oC
C) 100oC
D) 81oC
B)
Dry Heat
160 degrees C (320 degrees F) for 2 hours
OR 170 degrees for 1 hour
- denatures proteins
Pros:
- doesn't corrode/dull instruments
Cons:
- ruins heat-sensitive materials; instruments must be DRY bc water interes w/
sterilization

34.The MOST important viral cause of gastroenteritis in children less than 2 old is
A) rhinovirus.
B) cytomegalovirus.
C) coxsackievirus.
D) echovirus.
E) rotavirus.
E) Rotavirus
"Repeat-o-virus"; a REOVIRUS= dsRNA
ROTA= Right Out of The Anus
*** MOST imp global cause of infantile gastroenteritis
- acute diarrhea, esp in day-care centers, kindergartens during winter

35.Rubella and toxoplasmosis are similar in that both


A) are prevented by vaccinations.
B) are potentially teratogenic.
C) can not be treated with antibiotics.
D) primarily affect the elderly.
E) have non-human animal reservoirs
Maternal Infection Teratogens:
TORCH complex
1. Toxoplasmosis- parasitic disease caused by protozoan Toxoplasma gondii
- primary host is cat
- usually from infected meat or eating infected cat feces
- rarely causes symptoms in healthy people--> many enter latent phase
- acute phase- flu-like
- immunocompromised: encephalitis, neurologic, heart and liver problems
- Most infants who are infected while in the womb have no symptoms at birth
but may develop symptoms later in life
2. other agents
3. Rubella
4. CMV
5. HSV

36.The apical cytoplasm of active serous glandular cells is typically filled with
which of the following?
A) Abundance of mitochondria
B) Abundance of lipid droplets
C) Abundance of ribosomes
D) Abundance of zymogen granules
E) Large amount of DNA
D) abundance of zymogen granules
- basal processes (closest to lumen) are rich w/ MITOCHONDRIA- provide E
for ion transport

37.What is the composition of saliva?


- hypotonic
- enzymes: 1) amylase 2) lysozyme (antibacterial
- antibodies (IgA mostly)
- inorganic ions
- serous cells secrete fluid ISOTONIC w/ plasma

38.Which of the following BEST characterizes the alveolar mucous membrane?


A) Separated from the gingiva by the free gingival groove
B) Has no melanocytes
C) Appears red due to high vascularity and thinness of epithelium
D) Well developed epithelial ridges
E) Firmly bound to underlying bone
C)

39.The majority of cases of pharyngitis are caused by


A) hemophilus influenzae.
B) herpes simplex virus.
C) Staphylococcus aureus.
D) alpha-hemolytic streptococci.
E) a variety of viruses.
E

40.A fungus that causes systemic disease, most commonly of the lungs, and is
characterized by its production of tuberculate chlamydospores in culture is
A) Actinomyces israelii.
B) Microsponum cants.
C) Mycoplasma hominis.
D) Histoplasma capsulatum
E) Leptospira pomona.
D)
4 Systemic Mycoses:
- dimorphic fungi (mold in soil, yeast in tissue) except coccidiodomycosis

- Mold= cold; Heat=yeast


- all can cause PNEUMONIA and DISSEMINATE
- SYSTEMIC MYCOSES mimics TB (granulomas)
Tx: fluconazole or ketoconazole
1. Histoplasmosis- causes pneumonia
- Mississippi and Ohio R. Valleys
- bird or bat droppings
- intracellular (tiny yeast inside macrophages)
*** "Histo"="histiocyte" tissue macrophage
2. Blastomycosis= Big, Broad, Budding
3. Coccidiomycosis- California
- pneumonia and meningitis
- SPHERULE filled w/ endospores
- valley fever

41.Each of the following is a risk factor in atherosclerosis EXCEPT one. Which


one is this EXCEPTION?
A) Diabetes mellitus
B) Heredity
C) Hyperlipoproteinemia
D) Hypertension
E) Alcoholism
E) Alcoholism
Risks for atherosclerosis:
1. Smoking
2. Men and postmenopausal women (low estrogen)
3. HTN
4. heredity
5. nephrosclerosis (secondary HTN)
6. Diabetes
7. Hyperlipidemia (higher LDL= bad)
- most common sites:

- CAD (heart)
- abdominal aorta

42.Cementum differs from dentin in that cementum


A) contains some elastic fibers, whereas dentin contains only collagenous
fibers.
B) contains more inorganic material than dentin.
C) is not formed following eruption of the tooth.
D) can contain cells, whereas dentin contains cells as well as cell processes.
E) is produced by cells of the periodontal ligament, but dentin is produced by
pulp cells
E)
B is incorrect because dentin is 70% inorganic and enamel is 55% inorganic
(mineralized)

43.Excessive deposits of copper in liver cells, degenerative changes in the brain,


and a greenish-brown ring at the outer margin of the cornea characterize which
of the following?
A) Diabetes mellitus
B) Tay-Sachs disease
C) Phenylketonuria
D) Wilson's disease
E) Galactosemia
Wilson's Disease
- autosomal recessive- mutations in wilson disease protein
- Cu accumulates in tissues
- hepatolenticular degeneration
- liver disease
- neurological symptoms
-Cu in LIVER and BRAIN

44.Which of the following represents the major pathway for metabolism of


excessive intraneuronal free norepinephrine?
A) Hydroxylation by monoamine oxidase
B) Hydrolysis by cholinesterase
C) Hydroxylation by dopamine beta hydroxylase
D) Methylation by catechol-0-methyl transferase
E) Deamination by monoamine oxidase
D)

45.A subject consumes 250 ml of oxygen per minute with a tidal volume of 400 ml
and a respiratory rate of 18 per minute. Which of the following represents this
subject's respiratory minute volume in liters?
A) 5.4
B) 7.2
C) 10.0
D) 1.8
E) 4.5
B. 7.2
respiratory minute volume= 0.4 L x 18 breaths/min= 7.2

46.Which of the following types of tissues can be demonstrated on the posterior


slope of the articular eminence?A) Fibrocartilage
B) Fibrous connective tissue
C) Elastic cartilage
D) Hyaline cartilage
E) Articular cartilage
B) fibrous connective tissue

47.MOST fluid reabsorption by the kidney occurs in which of the following?


A) Proximal tubule
B) Descending loop of Henle
C) Distal tubule
D) Collecting duct

E) Ascending loop of Henle


A) Proximal Tubule

48.A hemorrhagic tendency is seldom seen in which of the following conditions?


A) Hepatic insufficiency
B) Secondary thrombocytopenia
C) Acute leukemia
D) Renal insufficiency
E) Scurvy
o A) hepatic insufficiency- bc it makes clotting factors
o B) low platelets= increase bleeding time
o C) acute leukemia= pancytopenia; low platelets increase bleeding time
o E) Scurvy- Vit C deficiency; get bleeding esp gums

49.The finding of yeast cells and chlamydospores in the oral mucosa suggest
which of the following?
A) Histoplasma capsulatum
B) Trichophyton mentagrophytes
C) Cryptococcus neoformans
D) Candida albicans
E) Blastomyces denmatitidis
D) Candida Albicans
- chlamydospore- thick-walled big resting spore of several kinds of fungi. It is
the life-stage which survives in unfavourable conditions, such as dry or hot
seasons.

50.Streptococcus pyogenes, group A, is subdivided into specific antigenic types


principally on the basis of immunologic differences in its
A) streptolysin 0.

B) hyaluronic acid capsule.


C) C polysaccharide.
D) M protein.
E) streptolysin S.
D) M Protein

51.Which of the following are MOST antigenic?


A) Nucleic acids
B) Proteins
C) Carbohydrates
D) Haptens
E) Lipids
B) Proteins
Hapten- "half an antigen"- a small molecule that can elicit immune response
only when attached to a larger carrier (like a protein)

52.Which of the following represents a DNA gyrase inhibitor with a broad


spectrum of activity?
A) Ciprofloxacin
B) Polymyxin
C) Rifampicin
D) Ethambutol
E) Cycloserine
A) Ciprofloxacin/Ofloxacin
- 2nd gen Fluoroquinolone class
- It kills bacteria by interfering with the enzymes that cause DNA to rewind
after being copied, which stops DNA and protein synthesis (DNA
gyrase/topoisomerase)
- Bactericidal
- AEROBIC/facultative gram + and - and mycobacteria; NOT ANAEROBES

53.A 6-year-old boy has minute white specks on the oral mucosa adjacent to his
first molars. A bluish-red ring surrounds these spots. He appears to have a cold
and his eyes are red and runny. There is a blotchy reddish rash behind his ears

and on his face. This child has which of the following?


A) MeaslesB) ChickenpoxC) Scarlet feverD) Eczema
A) Measles/Rubeola- a Paramyxovirus
- Koplik spots- red spots with white centers on buccal mucosa
- rash
3 C's of Measles:
1. cough
2. coryza- inflammation of mucous membranes
3. conjunctivitis

54.What condylar movement is performed as the mandible moves from a pure


protrusive movement from maximum intercuspal position to a maximum
protruded position?
A) Hinge
B) Medial and forward
C) Rotation
D) Translation
D) Translation

55.The absolute refractory period of a nerve action potential is determined by the


duration of which of the following?
A) Sodium inactivation gate closure
B) Potassium activation gate opening
C) Potassium inactivation gate closure
D) Sodium activation gate opening
A)
3 phases of Na+ gate:
- deactivated (closed)- blocked on their intracellular side by an "activation
gate", which is removed in response to stimulation that opens the channel.
- activated (open)
- inactivated (closed)
- local anesthesia favors activated and inactivated states--> rapidly firing

56.In an ideal intercuspal relation, the oblique ridge of the maxillary first molar
opposes which structure of the mandibular molar?
A) The developmental groove between the mesiofacial and distofacial cusps of
the first
B) The developmental groove between the distofacial and distal cusps of the
first
C) The interproximal area between first and second
D) The developmental groove between the mesiolingual and distolingual cusps
of the first
B)

57.In anemia resulting from drug-induced bone marrow suppression, the


peripheral blood smear shows erythrocytes to be
A) hyperchromic - macrocytic.
B) normochromic- microcytic.
C) hypochromic - microcytic.
D) normochromic- normocytic.
E) hypochromic - normocytic.
D

58.In protrusive movement, the mandibular canines in a Class II occlusal


relationship articulate with which of the following maxillary teeth?
A) Lateral incisors only
B) Canines and first premolars
C) Canines only
D) Canines and lateral incisors
D) Canines and lateral incisors

59.In an ideal intercuspal relation, the mesiofacial cusps of the mandibular second
molars contact the maxillary molars in (on) the
A) central fossae of the second.
B) lingual embrasures between the first and second.
C) mesial marginal ridges of the second.
D) distal fossae of the second.

E) facial embrasures between the first and second.


C)

60.The growth rate in the cartilage of the epiphyseal plate of a long bone is
markedly retarded when there is a lack of hormone from which of the
following?
A) Testes
B) Hypophysis
C) Islets of Langerhans
D) Adrenals
E) Parathyroids
B) hypophysis= pituitary gland!
- adenohypophysis= anterior pituitary
-->B-FLAT (basophophils) and GH, Prolactin, endorphins (acidophils)
- neurohypophysis= posterior pituitary
--> oxytocin and ADH/vasopressin

61.Regulation of each of the following mechanisms is associated with the


hypothalamus EXCEPT one. Which one is this EXCEPTION?
A) Carbohydrate metabolism
B) Body temperature
C) Pupillary diameter
D) Sleep
E) Water balance
C)

62.Which of the following is an antimicrobial agent that primarily inactivates


cellular DNA
A) Chlorhexidine
B) Alkaline glutaraldehyde
C) Phenols
D) Ethylene oxide

E) 70 percent isopropyl alcohol


D) Ethylene oxide gas
- sterilization method
- sterilization is slow 10-16 hours
- inactivates DNA and proteins
Pros:
- used mostly in HOSPITALS for heat-sensitive materials
Cons:
- long time
- VERY TOXIC and flammable; need good ventilation
A. chlorhexidine- inhibits plaque gingivitis (most potent inhibitor)
- Peridec, Perioguard
- stains teeth, alters taste perception
B. Glutaraldehyde (2%)- sterilization
- alkylates nucleic acids and proteins
- takes 10 hours--> used for respiratory equipment
- most potent CHEMICAL germicide
- associated w/ HYPERSENSITIVITY

63.Which of the following BEST describes the Curve of Spee?


A) The composite arrangement of the facial crown surface heights of contour of
all the teeth in any quadrant
B) The anterior-posterior curvature of the occlusal surfaces of the teeth, as seen
in a facial view
C) The inclination of the teeth in relation to the vertical long axis of the body
D) The spherical configuration of the composite arrangement of the occlusal
surfaces and incisal edges of the teeth of both dental arches
E) The facial-lingual curvature resulting from the facial cusps being the longest
in the mandibular arch, and the lingual cusps being the longest in the maxillary
arch
B)

64.Which of the following has a high affinity for binding calcium and collagen in
the calcifying matrix?

A) Amelogenin
B) Fibronectin
C) Osteogenin
D) Osteonectin
E) Calcitonin
D) Osteonectin
- glycoprotein in bone that binds calcium and has high affinity for collagen
- secreted by osteoblasts during bone formation
- vital in bone mineralization, cell matrix interactions, collagen binding
- increases activity of matrix metalloproteinases (imp for invading cancer cells
in bone)
- overexpressed in many cancers (e.g. breast, prostate, colon)

65.In a Class II occlusal relationship, the tip of the facial cusp of a mandibular first
premolar lies directly below the contacting area between which maxillary
teeth?
A) First and second premolars
B) Second premolar and first molar
C) Canine and first premolar
D) Canine and lateral incisor
A

66.Which of the following represents a striated muscle that contains transverse


tubules, a slow rate of calcium sequestration, and is inhibited by acetylcholine?
A) Multi-unit smooth
B) Cardiac
C) Single unit smooth
D) Skeletal
B)
T-tubules are less developed
- slow rate of Ca2+ sequestration to ensure ventricles can fill up
- Ach parasympathetics decrease its activity
- sympathetics (Epinephrine) increases activity

67.Which of the following causes activation of the pyloric pump, relaxation of the
pylorus, and contraction of the lower esophageal sphincter?
A) Pepsinogen
B) Gastrin
C) Acetylcholine
D) Cholecystokinin
E) Secretin
B) gastrin
- increases HCl secretion; stimulates parietal cells
- increases growth of gastric mucosa
- increases gastric motility
- increased by stomach distention, amino acids, peptides, vagal stimulation
- decreased by stomach pH <1.5
Ach- from vagal stimulation stimulates gastric parietal cell

68.Hypersensitivity to M. Tuberculosis is manifested by which of the following?


A) Langhans' giant cells
B) Spreading of the initial focus
C) Exudation
D) Epithelioid cells
E) Necrosis
E) necrosis

69.Dentinal tubules are S-shaped in the crown of the tooth due to the
A) formation of peritubular dentin.
B) calcification pattern of maturing dentin.
C) epithelial diaphragm.
D) crowding of odontoblasts.
E) incremental pattern.
D)

70.Which group of fibers of the periodontal ligament is the first to offer resistance
to movement of the tooth in an occlusal direction?

A) Horizontal
B) Apical
C) Oblique
D) lnterradicular
E) Alveolar crest
B) apical
- apical fibers are at the apex of the root

71.In a newly erupted tooth, the junction between tooth surface and the crevicular
epithelium consists of which of the following?
A) Basal lamina-like structure between enamel and epithelium
B) Keratin fibers, running from the epithelium deeply into the enamel
C) Basal lamina-like structure between dentin and epithelium
D) Interstitial crevicular fluid
E) Basal lamina-like structure between cementum and epithelium
A) Basal-lamina like structure btwn enamel and epithelium- forms epithelial
attachment
- secreted by epithelial cells; epithelial cells on top of an ECM often confused
as basement membrane
- Basal lamina:
1. lamina lucida
2. lamina densa- electron dense
Basement membrane:
1. lamina lucida
2. lamina densa
Lamina reticularis

72.Which of the following is a function of the enterogastric reflex?


A) Decreases motility of the stomach
B) Increases motility of the ileum
C) Increases gallbladder emptying
D) Decreases the opening of the gastroesophageal sphincter
E) Increases motility of the esophagus
A)
Enterogastric reflex- one of 3 reflexes in GI tract

+ stimulation:
- acid in duodenum
- stomach pH < 1.5
Results in:
- inhibit gastrin from G-cells in antrum of stomach
- inhibit gastric motility and HCl secretion
- purpose: shuts off stomach motility when there is chyme in duodenum

73.Vascular smooth muscle relaxes in response to


A) adenosine.
B) norepinephrine.
C) hyperoxia.
D) vasopressin.
E) angiotensin
A) adenosine
- G-protein coupled receptors
adenosine imp in:
1. E transfer (e.g. ATP,ADP)
2. signal transduction (e.g. cAMP)
3. inhibitory NT- promote sleep and arousal
4. causes endothelial dependent relaxation of smooth muscle and is found
inside artery walls

74.Buerger's Disease
aka Thomboangiitis obliterans
-recurring inflammation and thrombosis of small and medium arteries of hands
and feet
- main symptom= PAIN in affected areas
- complications= ulcerations and gangrene of extremities--> need amputation
- strongly associated with tobacco products, esp smoking

Etiology:
- MEN in 20-40s (like Warthin's Tumor)

75.A 53-year-oid patient has an indurated, chronic ulcer near the inner canthus.
The MOST likely diagnosis is
A) adenocarcinoma of the lacrimal duct.
B) verruca vulgaris.
C) squamous cell carcinoma.
D) malignant melanoma.
E) basal cell carcinoma.
o E) basal cell carcinoma- most common skin malignancy
o - never metastasizes
o - malignant bc destroys surrounding tissues

76.The oral lesions of herpangina can often be distinguished from hand-foot-andmouth disease by which of the following?
A) Their gradual confluence
B) Their physical appearance
C) Their intraoral locations
D) Their density and number
C) Coxsackie A virus- picoRNA, ssRNA, non-enveloped
Herpangina- self limited
- tonsillar pillars and soft palate, pharyngitis
- malaise, fever, dysphagia
- outbreaks in summer: fecal/oral, contaminated saliva
Hand-foot-mouth
- airborne/ fecal-oral
- oral lesions favor palate, tongue, buccal mucosa
- mostly in children
- fever, malaise, sore mouth, lymphadenopathy

77.The submucosa is present in each of the following EXCEPT one. Which one is
this EXCEPTION?A) Jejunum
B) Colon
C) Duodenum
D) Stomach
E) Gallbladder
E) Gall Bladder
- non-vital organ
- stores bile
Histology:
- simple columnar epithelial lining
- Aschoff's recesses- pouches inside the lining
- lamina propria- connective tissue
- smooth muscle (muscularis externa)- contracts in response to CCK from
duodenum to release bile
- NO SUBMUCOSA btwn mucosa and adventitia; just thin muscular layer that
prevents infection

78.Which of the following is the site of oxidative phosphorylation in bacteria?


A) Cell membrane
B) Mitochondrion
C) Nucleus
D) Ribosome
E) Cytoplasm
A) cell membrane

79.Each of the following develops as an outpocketing of the gut tube EXCEPT


one. Which one is this EXCEPTION?
A) Liver
B) Gallbladder
C) Pancreas
D) Lung
E) Spleen

E) Spleen
- like thymus, contains only EFFERENT vessels
- forms w/in and from the dorsal mesentery (DERIVED FROM
MESENCHYMAL tissue)
- mesenchyme- reticular connective tissue derived from all 3 germ layers
- can dev into cartilage, lymphatic system, circulatory, connective tissue
- holds reserve of RBCs in case of hemorrhagic shock
- recycles iron
- removes old RBCs
white pulp
- synthesis Ab's
- removes Ab-coated bacteria and Ab-coated blood cells
- removal leads to increase infections
*** All the rest are derived from gut tube (endoderm)

80.Which of the following bacterial exotoxins converts plasminogen to plasmin?


A) Plasmalysin
B) Hyaluronidase
C) Streptokinase
D) M-protein
E) Coagulase
C) streptokinase

- inexpensive blood-clotting medication

81.Which of the following conditions predisposes to lung cancer by causing


squamous metaplasia of bronchial epithelium?
A) Bronchial asthma
B) Pulmonary emphysema
C) Bronchiectasis
D) Chronic bronchitis
E) Bronchial carcinoid
o D) Chronic Bronchitis (COPD)--> Cigarette smoking
o - increase lung cancer- squamous metaplasia from chronic inflammation
o - blue bloaters- early cyanosis; late dyspnea
o - productive cough, noisy chest
o - chronic cough 3 months in 2 yrs
o - increase reid index- mucosal gland thickness: smooth muscle

82.Which of the following represents the MOST frequent cause of a clinically


palpable breast mass in an adult woman?
A) Intraductal papilloma
B) Adenocarcinoma
C) Fibrocystic disease
D) Sarcoma
E) Fibroadenoma
C) fibrocystic disease- normal collagen buildup; benign
- fibroadenoma- most common in younger than 25 yrs
- Invasive ductal= most common malignant breast tumor; worst and most
invasive
- firm, fibrous mass, small, glandular, duct-like cells

83.Which of the following conditions increase the risk of developing


osteosarcoma?
A) Osteoblastoma
B) Osteogenesis imperfecta
C) Osteitis deformans
D) Osteoporosis
E) Osteomalacia
C) Osteitis deformans= Paget's disease
- uncontrolled osteoclast/osteoblast activity
- increase in size of head; enlarged and deformed bones
- bone is weak - arthritis, fractures
- Men>women; less than 40s
- RARELY results in OSTEOGENIC SARCOMA
a) osteoblastoma- benign tumor found in vertebral column - trabeculae and
woven bone surrounded by osteoblasts
b) osteogenesis imperfecta- Type I collagen deficiency
- autosomal dominant- aa substitituion of glycine by a bulkier one
- blue sclera, bone fractures, hearing loss (bony ossicles malformed), dentin
malformation
d) osteoporosis- esp in post-menopausal women (decrease in estrogen);
decrease in bone density
e) osteomalacia- softening of bones; vit D deficiency in adults

84.Neurofibromatosis type I is characterized by which of the following


conditions?
A) Perioral melanotic freckles
B) Medullary carcinoma of thyroid
C) Multiple pigmented macules of the skin
D) Multiple mucosal neuromas
E) Development of visceral carcinoma
C) Cafe au laix macules
Perioral melanotic macules- Addison's and Peutz-Jegher
Neurofibromatosis- Autosomal Dominant tumors of melanocytes and schwann
cells (derived from neural crest)
- harmless or cause damage by compressing nerves/tissue

1. Type I- Lisch Nodules, neural tumors, cafe aux laix, skeletal disorders,
pheochromocytosis; chromosome 17
2. Type II- characterized by ACOUSTIC SCHWANNOMAS (deafness)

85.Which of the following cancers typically arises in teenagers and young adults?
A) Multiple myeloma
B) Chronic lymphocytic leukemia
C) Wilms' tumor
D) Angiosarcoma
E) Ewing's sarcoma
E) Ewings (t 11,22)
- onion peel appearance
- early metastasis
- v. responsive to radiotherapy
- SMALL BLUE cell malignant tumor
- most common in boys <15 yrs old
C) Wilm's tumor
- most common renal malignancy of early childhood (age 2-4)
- huge palpable mass

86.Of the primary maxillary teeth, the cervical ridge would stand outMOST
prominently as a distinct entity on which surface of which molar?
A) Second molar, Distofacial surface
B) Second molar, Distolingual surface
C) First molar, Mesiofacial surface
D) First molar, Distofacial surface
E) Second molar, Mesiolingual surface
C)

87.Following eruption and initial occlusal contact in the oral cavity, a tooth will
continue to erupt in order to compensate for occlusal wear. In response to this
continuous eruption, which of the following is deposited at the apex of the
root?

A) Reparative dentin
B) Primary dentin
C) Cellular cementum
D) Acellular cementum
E) Secondary dentin
C) Cellular cementum
- located at the 1/2-1/3 apical portion of root
- cementum lacks vascularization
- Acellular cementum is the enamel located at the lower part of then enamel
(more cervical)

88.Which of the following teeth is the LEAST likely tooth to have a bifurcation of
the root?
A) Mandibular central incisor
B) Maxillary central incisor
C) Maxillary second premolar
D) Mandibular canine
E) Mandibular lateral incisor
B

89.Each of the following is characteristic of herpes zoster EXCEPTone. Which


one is this EXCEPTION?
A) More common in individuals who are immunocompromised.
B) Only occurs in an individual having a latent VZV infection.
C) Unilateral
D) Occurs repeatedly in immunocompetent patients.
E) Usually involves 1-3 dermatomes.
D)
VZV
Children
- Chickenpox
Adults
- zoster
- posterherpetic neuralgia

90.Which of the following is the lobe of the cerebral hemisphere that lies in the
middle cranial fossa?
A) Parietal
B) Occipital
C) Frontal
D) Temporal
D)
frontal- anterior fossa
occipital- posterior cranial fossa
anterior and middle cranial fossa separated by lesser wing of sphenoid bone
middle and posterior cranial fossa separated by petrous part of temporal bone

91.MOST cases of choriocarcinoma are discovered by the appearance of a rising


titer in blood or urine of which of the following?
A) Human chorionic gonadotropin
B) Alkaline phosphatase
C) Acid phosphatase
D) Alpha-fetoprotein
E) Carcinoembryonic antigen
A) HcG
choriocarcinoma- aggressive, malignant cancer of placenta
- early hematogenous spread to LUNGS
- v. sensitive to chemotherapy

92.The apex of the horizontal plane Gothic-arch tracing represents which of the
following positions?
A) Centric relation
B) Rest position
C) Maximum opening
D) Lateral protrusive

o
o A)

93.Which of the following statements is correct with respect to mastication of


food?
A) Indigestion is generally related to inadequate mastication of food.
B) Proper incisor contact is a requisite for masticatory efficiency.
C) Food is masticated primarily in lateral contacting movement.
D) Food is masticated primarily in a border movement.
E) An ideal occlusion is a requisite for masticatory efficiency.
C)

94.A mature ovum is expelled from which of the following?


A) Corpus albicans
B) Graafian follicles
C) Corpus hemorrhagicum
D) Corpus luteum
B)
1. graafian follicle ruptures to release mature ovum
2. corpus hemorrhagicum immediately after rupture
3. Corpus luteum
4. corpus albicans= degenerated corpus luteum

95.TNF-Beta
o a lymphokine cytokine that kills virally infected cells by producing holes
in cell membrane
o - released by Cytotoxic T8 cells

96.Calcium dipicolinate is present in large amounts in


A) mitochondria.
B) ribosomes.
C) f1agella.
D) spores.
E) mesosomes.
D

97.Each of the following statements describes a contributor to the initiation of


mineralization of bone EXCEPT one. Which one is this EXCEPTION?
A) Release of acid phosphatase by osteocytes trapped in lacunae
B) Holes or pores in collagen fibers
C) Alkaline phosphatase activity in osteoblasts and matrix vesicles
D) Degradation of matrix pyrophosphate to release an inorganic phosphate
group
E) Release of matrix vesicles by osteoblasts
A

98.Before swallowing can be initiated, afferent information must be received


A) from mucosal mechanoreceptors, indicating the consistency of a soft bolus
of food.
B) from pharyngeal taste receptors, indicating the consistency of a soft bolus of
food.
C) in a lesser amount form nociceptors, indicating a soft bolus of food.
D) from muscle spindles, indicating the consistency of a soft bolus of food.
A) CONSISTENCY!

99.Glucose can be made from each of the following substancesEXCEPT one.


Which one is this EXCEPTION?A) Fructose
B) Glycerol
C) Pyruvate
D) Lactate
E) Acetyl CoA
E)

100.
The movement in the upper compartment of the temporomandibular
joint is rotation and translation. The movement in the lower compartment is
translation.
A) The first statement is TRUE, and the second isFALSE.
B) The first statement is FALSE, and the second isTRUE.
C) Both statements are FALSE.
D) Both statements are TRUE.
C)
ROTATION= hinge movement
upper compartment= translation
lower compartment= hinge-type rotation

101.
Juxtamedullary nephron vs. cortical nephron
- differ mostly by the length of the loop of Henle
- named based on location of the renal corpuscle (glomerulus + Bowman's
capsule)
1. juxta- more rare; responsible for creating osmotic gradient in medulla

102.
Atropine prevents the secretion of saliva resulting from stimulation of
the chorda tympani because it
A) prevents release of acetylcholine by sympathetic postganglionic fibers.
B) prevents release of acetylcholine in autonomic ganglia.
C) prevents the action of acetylcholine on the secreting cells.

D) denatures the salivary gland proteins.


- competitive antagonist for the muscarinic receptor so it doesn't allow Ach to
bind

103.
If the anticodon on transfer-RNA is 5'ACG3', then which of the
following is its corresponding codon on messenger-RNA?
A) 5' UAG 3
'B) 5' CGT 3'
C) 5' CGU 3'
D) 5' UGC 3'
E) 5' TGC 3'
C

104.
Cigarette smoking has been etiologically linked to each of the following
cancers EXCEPT one. Which one is this EXCEPTION?
A) Colon
B) Oral cavity
C) Esophagus
D) Urinary bladder
E) Lung
A)

105.
In the intercuspal position, which of the following anterior teeth has the
potential of contacting both anterior and posterior antagonists?
A) Maxillary canine
B) Mandibular canine
C) Mandibular lateral incisor
D) Maxillary lateral incisor
A

106.
On the crown of the primary maxillary first molar, the mesial surface
normally
A) exhibits a straight cervical line.
B) is wider buccolingually at the occlusal table than at the cervical third.

C) is larger than the distal surface of the same tooth.


D) exhibits a concave lingual outline.
E) is wider occlusocervically than buccolingually.
C

107.
Which of the following factors MOST influences the lingual concavity
of the maxillary anterior teeth and groove direction of the posterior teeth?
A) Angle of the eminence
B) Sideshift of the mandible (Bennett movement)
C) Anterior guidance
D) Direction of movement of the rotating condyle
E) Curve of the occlusion
B

108.
Each of the following is attributable to hepatic failure EXCEPT one.
Which one is this EXCEPTION?A) Hypoalbuminemia
B) Mallory bodies
C) Tremor
D) Spider telangiectasia
E) Gynecomastia

o B) Mallory Body
o - alcoholic hyaline inclusion in cytoplasm of liver cells
o - most common in alcoholic hepatitis and alcoholic cirrhosis

109.
Which main product of protein nitrogen metabolism is found in human
urine?
A) Ammonia
B) Creatinine
C) Urea
D) Creatine
E) Uric acid
C) urea

110.
During the life span of a multi rooted tooth, dentin continues to form
MOSTrapidly at which of the following locations?
A) Facial and lingual walls of the pulp chamber
B) Mesial and distal walls of the pulp chamber
C) Within the root canals
D) Floor and roof of the pulp chamber
E) At the dentinoenamel junction
D

111.
A patient reports breaking a lower right central incisor while eating a
cookie. Which of the following mandibular movements was the patient MOST
likely making when this occurred?
A) Retrusive
B) Right working
C) Protrusive
D) Left working
E) Straight closure
C

112.
Which of the following types of bronchogenic carcinoma is MOST
commonly associated with paraneoplastic syndrome?
A) Adenocarcinoma
B) Mesothelioma
C) Small cell carcinoma
D) Large cell carcinoma
C)
Paraneoplastic syndrome
- disease or symptom that is the consequence of cancer in the body, but is not
due to local presence of cancer cells
Small cell- ectopic production of ACTH or ADH
- can lead to Lambert-Eaton syndrome (auto-Ab's against Ca2+ channels)

113.
The articular disc of the temporomandibular joint consists of
A) an outer layer of mesothelium and an inner core of hyaline cartilage.
B) an outer synovial layer and an inner fibrous layer.
C) hyaline cartilage.
D) loose fibrous connective tissue.
E) dense fibrous connective tissue.
E

114.
In a CUSP TO FOSSA contacting relationship in intercuspal position,
the maxillary first premolar is MOST likely to articulate with which of the
following mandibular teeth?
A) Lateral incisor and canine
B) First premolar and second premolar
C) Canine and first premolar
D) Lateral incisor
E) First premolar
E)
Because of the mesial swing of the lingual cusp
- nothing occludes in the mesial fossas of the mandibular teeth

115.
The OLDEST enamel in a fully erupted first molar is located at the
cervix of the tooth.
A) dentinoenamel junction underlying a cusp.
B) oral surface of a cusp.
C) dentinoenamel junction underlying a fissure.
D) mesial and distal surfaces of the tooth.
C)

116.
Which of the following represents the structure in the maxillary alveolar
bone that maxillary premolar roots occasionally penetrate?
A) Zygomatic arch
B) Antrum
C) Mandibular fossa
D) Frontal sinus
E) Nasal septum
B)

117.
Which of the following is MOST likely to precede an impending
atherosclerotic cerebral infarction?A) Myocardial infarction
B) Ruptured berry aneurysm
C) Pulmonary embolus
D) Transient ischemic attacks
E) Angina pectoris
D

118.
The biologically active conformation of trimeric G-proteins requires
A) the alpha-subunit to bind GTP.
B) the hydrolysis of BetaGamma-subunits.
C) the alpha-subunit to phosphorylate downstream targets
.D) the alpha-subunit to bind GDP.
A

119.
Which of the following organs secretes insulinlike growth factor 1 (IGF1), and what stimulates this secretion?A) Kidney, increasing growth hormone in

the blood
B) Liver, increasing growth hormone in the blood
C) Liver, increasing TSH in the blood
D) Anterior pituitary, increasing IGF-RH in the blood
E) Anterior pituitary, increasing GRH in the blood
B)
IGf-1 is made in liver and stimulated by GH (from anterior pituitary)
- binds to IGF-R (tyrosine kinase)
- stimulates body growth

120.
In which of the following structures are glycoproteins assembled for
extracellular use?
A) Golgi apparatus
B) Rough endoplasmic reticulum
C) Smooth endoplasmic reticulum
D) Nucleolemma
E) Polyribosomes
o A) GLYCOproteins
o - protein made in RER, but glycosylated/modified in Golgi for
extracellular use

121.
Ingestion of which of the following MOST markedly DECREASES
gastric emptying?
A) Carbohydrates
B) Fats
C) Water
D) Minerals
E) Proteins
B)
- decrease gastric emptying bc CCK (need more time to break down the fats)

122.
A patient bites down rapidly on an unexpected hard surface while
chewing. Cessation of motor unit recruitment in jaw closing muscles is caused
by stimulation of
A) periodontal mechanoreceptors.
B) nociceptors in the dental pulp.
C) mucosal mechanoreceptors.
D) muscle spindles.
A)

123.
Edema as a result of a hemodynamic disorder commonly occurs in each
of the following locations EXCEPT one. Which one is this EXCEPTION?
A) Peritoneal cavity
B) Lungs
C) Brain
D) Pancreas
E) Subcutaneous tissues
D) pancreas

124.
Inorganic crystals in enamel have their long axes parallel to the rods in
which of the following?
A) Tails of the rods and deviating increasingly from the centers of the bodies to
the periphery
B) Centers of the bodies of the rods only
C) Tails of the rods and deviating increasingly from the periphery of the bodies
to the centers
D) Bodies of the rods and deviating increasingly in the tails
E) Periphery of the bodies of the rods only
D)

125.
Which of the following temporomandibular joint ligaments restricts the
movement of the disc away from the condyle during function?
A) Sphenomandibular
B) Temporomandibular
C) Discal

D) Stylomandibular
C) Discal

126.
THe condyle on the laterotrusive side generally rotates about a
A) vertical axis and translates laterally.
B) sagittal axis only.
C) horizontal axis only.
D) horizontal axis and translates laterally.
A

127.
Which of the following teeth has its mesial marginal ridge located more
cervically than its distal marginal ridge?
A) Maxillary first molar
B) Mandibular first premolar
C) Primary mandibular first molar
D) Mandibular second molar
E) Maxillary first premolar
B

128.
The main route of calcium excretion from a normal human adult is
A) sweat.
B) feces.
C) urine.
D) tears.
E) saliva.
B) feces

129.
The first clinical findings in prostatic cancer are often the result of
metastasis to which of the following?A) Brain
B) Testes
C) Bone
D) Liver
E) Adrenal gland
C

130.
In the sulcular epithelium, small spaces between cells in the stratum
spinosum (prickle cell layer) are normally filled by
A) none of the above.
B) capillaries that course between cells to approach the free surface.
C) keratin.
D) a small amount of tissue fluid.
D

131.
Excluding the effect of drugs, apnea occurring after hyperventilation of
an anesthetized patient results from
A) increased hydrogen ion concentration.
B) decreased demand for oxygen.
C) increased cerebral blood flow.
D) decreased oxygenation of carotid bodies.
E) decreased carbon dioxide tension.
E.
apnea= temporary cessation of breathing

132.
Which of the following increase with age in dental pulp?
(a) Vascularity; (b) Size of the pulp chamber; (c) Number of collagen fibers; (d)
Calcification within the pulp
number of collagen fibers and calcification w/in pulp increase

133.
Sounds heard during systole in the antecubital space are produced by
A) turbulent blood flow through the artery.B) laminar blood flow through the
occluded artery C) closure of the aortic valve.D) closure of AV valves.
A) turbulent blood flow through artery
antecubital space= the front of the elbow
systole= contraction

134.
Normal hemoglobin concentration is about 15 gm./dl. blood and normal
arterial oxygen content is about 20 ml. 02/dl. blood. An anemic individual
breathing room air with a hemoglobin concentration of 10 gm./dl. blood is
expected to have
A) reduced arterial oxygen tension and normal arterial oxygen content.
B) reduced arterial oxygen tension and reduced arterial oxygen content.
C) normal arterial oxygen tension and normal arterial oxygen content.
D) normal arterial oxygen tension and reduced arterial oxygen content.
D)
Oxygen tension means oxygen partial pressure
- when you increase O2 pp, you decrease CO2 pp and vice versa
- partial pressure of O2 will stay the same in anemia, but O2 content will
decrease because less Hemoglobin

135.
It is possible to distinguish histologically between the stomach and the
duodenum because of the presence of
A) smooth muscle in the external musculature of the duodenum only.
B) simple columnar epithelium lining the stomach only.
C) submucosal glands in the duodenum only.
D) mucosal glands in the stomach only.
E) muscularis mucosa in the stomach only.
D

136.
Tropocollagen is a protein molecule found in
A) elastic and reticular fibers only.
B) collagen and reticular fibers only.
C) collagen, elastic and reticular fibers.
D) collagen and elastic fibers only
B) collagen and reticular fibers only

137.
Cells of the stratum granulosum of thick stratified squamous epithelium
characteristically contain
A) granules that are organelles. B) keratohyalin granules.C) melanin
granules.D) keratin granules.
B) keratohyalin granules

138.
At a muscle-tendon junction, the union is made by
A) a continuity of connective tissue sheaths of the muscle with those of the
tendon.
B) a special thickening of sarcoplasm that unites with collagenous fibrils of the
tendon.
C) an abundance of reticular fibers in the area of the junction.
D) myofibrils connecting with collagenous fibrils of the tendon.
A)

139.
The number of roots that are formed is determined by the
A) number of medial ingrowths of the cervical loop.
B) number of discontinuities developed in the root sheath.
C) thickness of the cervical loop.
D) number of root sheaths developed by the enamel organ.
A) # of medial ingrowths of the cervical loop--> 3 of roots
# of discontinuities developed in the root sheath--> determines ACCESSORY
CANALS

140.
Which of the following are mobilized when a cell produces an excessive
amount of protein?
A) All of the above B) Lipofuscin granulesC) LysosomesD) MitochondriaE)
Rough endoplasmic reticula
C) lysosomes to degrade the excess

141.
The alveolar bone proper (cribriform plate) usually consists of
A) lamellar bone only.B) bundle bone and lamellar bone. C) bundle bone
only.D) woven bone.
B) bundle bone and lamellar bone

142.
Enamel spindles are formed by
A) odontoblasts.B) cracks.C) hypoplastic rods.D) hypocalcified rods. E)

ameloblasts.
A) odontoblasts
enamel spindles- odontoblastic processes that extend into enamel (trapped by
enamel); can reach the surface and cause some sensitivity
enamel tufts- small enamel tufts that extend into enamel from CEJ
enamel lamellae- hypocalcified enamel defects- cell debris and enamel proteins
that extend to surface
- an area that bacteria can enter

143.
Which of the following laboratory results is diagnostic of acute
pancreatitis?
A) Elevated alkaline phosphatase.
B) Decreased serum amylase
C) Decreased albumin
D) Elevated serum lipase
D) elevated serum lipase

144.
A complication of peptic ulcer disease that accounts for the majority of
deaths is
A) malignant transformation.
B) obstruction from edema or scarring.
C) perforation.
D) surgical complication.
E) bleeding.
C) Perforation
90% caused by H. pylori
#1 complication is bleeding
#1 cause of death is perforation (peritonitis)
- can penetrate liver and pancreas

145.
A cross section at midroot of a permanent mandibular central incisor is
likely to show that the pulp cavity is
A) flattened mesiodistally.
B) flattened faciolingually.
C) divided into two canals.
D) shaped much the same as that of a maxillary central incisor.
E) almost round.
A) bc wider faciolingually

146.
Which of the following permanent teeth has its mesial marginal ridge
located more cervically than its distal marginal ridge?
A) Mandibular first molar
B) Maxillary second premolar
C) Mandibular first premolar
D) Maxillary first molar
E) Mandibular second premolar
F) Mandibular second molar
C

147.
In a normal, healthy mouth, the interdental papilla that is the shortest
cervicoincisally (cervico-occlusally) is between mandibular
A) second premolar and first molar.
B) central incisors.
C) central and lateral incisors.
D) first and second premolars.
E) lateral incisor and canine.
F) canine and first premolar.
A) 2nd premolar and 1st molar

148.
A patient with which of the following diseases is least likely to show
"clubbing" of fingers and cyanotic nail beds?
A) Congestive heart failure
B) Chronic pulmonary disease
C) Polycythemia

D) Systemic lupus erythematosus.


E) Congenital heart disease
o Associated w/
o - lung disease
o - heart disease
o - GI and hepatobiliary
o - hyperthyroidism

149.
In the intercuspal position, the cusp tip of the permanent maxillary
canine is in direct alignment with which anatomic feature of the mandibular
teeth?
A) Interproximal space
B) Facial embrasure
C) Distal ridge of the cusp of the mandibular canine
D) Mesial ridge of the facial cusp of the mandibular first premolar
E) Incisal embrasure
B.

150.
Pronounced developmental grooves are usually associated with
embrasures between permanent:
(a) maxillary central and lateral incisors; (b) maxillary lateral incisors and
canines; (c) maxillary canines and first premolars; (d) maxillary first and
second premolars; (e) mandibular canines and first premolars; (f) mandibular
first and second premolars; (g) mandibular second premolars and first molars;
(h) mandibular first and second molars.
A) (c) onlyB) (d) onlyC) (g) onlyD) (b) onlyE) (d) and (f)F) (a) onlyG) (c) and
(e)
G) C and E
- mesial marginal groove on max 1st premolar
- mesiolingual developmental grove on mand 1st premolar

151.

The drawing is a cross section at the cervical line of what permanent tooth?
A) Mandibular first premolar
B) Mandibular canine
C) Maxillary lateral incisor
D) Mandibular central incisor
D) mandibular central

152.
A primary mandibular second molar differs from a permanent
mandibular first molar in that a primary mandibular second molar
A) is larger in size.
B) has a form peculiar to itself.
C) has a larger occlusal surface.
D) has roots that are more divergent.
E) is darker in color.
D) bc the permanent premolar grows inbetween the roots

153.
The most common cause of a massive hematemesis in alcoholics is
A) esophageal varices.
B) peptic ulcer.
C) acute hemorrhagic pancreatitis.
D) acute gastritis.
E) Mallory-Weiss syndrome.
A) esophageal varices

154.

Viewed anteriorly, which diagram represents a chewing stroke?


4

155.

The sketch represents the


A) mesial view of a permanent mandibular third molar.
B) distal view of a permanent mandibular first molar.
C) distal view of a permanent mandibular second molar
.D) mesial view of a permanent mandibular first molar.
E) mesial view of a permanent mandibular second molar.
B

156.
In an ideal intercuspal position, the distolingual cusp of a permanent
maxillary first molar opposes the
A) lingual sulcus of the mandibular first molar.
B) facial embrasure between mandibular first and second molars.
C) mesial fossa of the mandibular second molar.
D) mesial marginal ridge area of the mandibular second molar.
E) central fossa of the mandibular first molar.
D

157.
Bacteria of which of the following genera have a limited range of
habitats in the oral cavity ?
(a) Treponema; (b) Bacteroides; (c) Actinomyces (d) Streptococcus
A and B
Can't Breath Air= anaerobic
Bacteriodes
Treponema= motile spirochete that is anaerobic
Actinomyces- anaerobic or facultatively anaerobic

158.
Features of the herpes simplex virus type 1 virion surface include
A) an envelope synthesized de novo.
B) none of the above.
C) an envelope acquired by budding through the nuclear membrane.
D) no envelope.

E) an envelope acquired by budding through the cytoplasmic membrane.


C)

159.

Assuming occlusion and alignment are normal, the arrows on the sketch
represent the path taken by the
A) lingual cusp of second premolar and the mesiolingual cusp of first molar.
B) cusp of canine and facial cusp of first premolar.
C) facial cusp of second premolar and the mesiofacial cusp of first molar.
D) facial cusps of premolars.
E) lingual cusps of premolars.
D

160.
In which of the following areas is the alveolar process the thinnest?
A) Facial to the maxillary second molars
B) Lingual to the maxillary central incisors
C) Lingual to the mandibular first molars
D) Lingual to the maxillary second molars
E) Lingual to the maxillary canines
F) Facial to the mandibular central incisors
F

161.
The protrusive pathway of mandibular cusps on maxillary posterior teeth
is toward the
A) lingual.B) None of the above C) mesial.D) distal.E) facial.
C mesial

162.
In the intercuspal position, which of the following anterior teeth has the
potential to contact both anterior and posterior antagonists?
A) Mandibular canine
B) Mandibular lateral incisor
C) Maxillary canine

D) Maxillary lateral incisor


C. maxillary canine

163.
Phage conversion is responsible for
A) ability of Rhizobium species to fix nitrogen.
B) production of B-galactosidase
C) transduction of bacteria.
D) production of erythrogenic toxin by Streptococcus pyogenes.
E) antigenic phase variation.
D)
Phage conversion= Lysogenic conversion
- when a temperate bacteriophage (virus that infects bacteria) induces a change
in phenotype of the bacteria infected--> increases the pathogenic capability of
the bacteria/host
- can later turn into lytic phase

164.
Calcification of the roots of the primary dentition is normally completed
at what age?
A) 7-8 years
B) 5-6 years
C) None of the above
D) 3-4 years
E) 1-2 years
D) 3-4 yrs
- roots are 50% calcified at time of eruption

165.
Which of the following permanent teeth is least likely to have a divided
pulp canal?
A) Maxillary first premolar
B) Mandibular central incisor
C) Mandibular lateral incisor
D) Mandibular first premolar

E) Maxillary central incisor


E) Maxillary central incisor

166.
Generally, the antibiotic of choice for prophylactic therapy covering
dental procedures in a patient with a heart valve abnormality who is allergic to
penicillin
is
A) lincomycin.
B) tetracycline.
C) erythromycin.
D) ampicillin.
E) streptomycin.
C) erythromycin
- a macrolide w/ antimicrobial spectrum similar to or slightly wider than
penicillin
- used for people w/ allergy to pen
- inhibits protein synthesis by bacteria by binding 50S ribosome
- *** CAUSES GI DISTURBANCES- DIARRHEA, NAUSEA, AB PAIN bc it
increases motilin secretion

167.
Most rickettsial diseases produce severe illness in humans because
rickettsiae
A) are primarily neurotropic and cause extensive damage in certain CNS
centers.
B) are particularly destructive to cutaneous tissues.
C) produce potent exotoxins.
D) are destructive for endothelial cells.
D) destructive for endothelial cells
Rickettsia
- motile, Gram (-), non spore-forming, highly pleomorphic bacteria
- OBLIGATE intracellular parasites that usually depend on growth and
replicated in endothelial cells of host
- susceptible to tetracycline
- carried as parasites by many ticks, fleas, lice

- Rocky Mountain Spotted Fever


- Rickettsia Typhi
- Rickettsia Prowezaki (lice)

168.
In the mandibular arch, the greatest lingual inclination of the crown from
its root is seen in a permanent
A) lateral incisor.B) canine.C) central incisor.D) third molar.E) first premolar.
E) 1st premolar

169.
The most acceptable conditions for complete sterilization by autoclaving
are
A) 15 minutes at 275 F. (135 C.) and 10 lbs. of pressure.
B) 20 minutes at 250 F. (121 C.) and 10 lbs. of pressure.
C) 20 minutes at 250 F (121 C) and 15 lbs. of pressure.
D) 5 minutes at 275 F. (135 C.) and 15 lbs. of pressure.
C)

170.
In an ideal intercuspal position, a permanent mandibular lateral incisor
opposes the:
(a) distal marginal ridge of a maxillary central incisor;
(b) mesial marginal ridge of a maxillary lateral incisor;
(c) distal marginal ridge of a maxillary lateral incisor;
(d) mesial marginal ridge of a maxillary canine;
(e) maxillary lateral incisor 2-3 mm. cervically to the incisal edge;
(f) maxillary lateral incisor at the incisal edge.
A) (c), (d) and (e) B) (a), (b) and (f)C) (a) and (b) onlyD) (a), (b) and (e)E) (b)
only
a,b,e

171.
The lingual cusp of a mandibular first premolar contacts the
A) mesial incline plane of the lingual cusp of a first premolar
.B) mesial marginal ridge of a maxillary first premolar.
C) cingulum of a maxillary canine.

D) None of the above


E) lingual embrasures of a maxillary canine and a maxillary first premolar.
D) none of the above! the lingual cusp of 1st molar is nonfunctional (acts most
closely to the cingulum of a canine)

172.

Which of the following is characterized by a collapse of alveoli?

A) Emphysema
B) Atelectasis
C) Pneumonia
D) Bronchiectasis.
E) Empyema
B) Atelectasis
- lack of gas exchange w/ lung alveoli due to alveolar collapse or fluid
consolidation
- may occur as post operative complication OR surfactant deficiency
- in premature neonates: causes infant respiratory distress syndrome
cough, but not prominent
chest pain
breathing difficulty
low oxygen saturation- amount of Hb saturated w. O2
pleural effusion (transudate type)
cyanosis (late sign)
increased heart rate

173.
Infants are routinely immunized against diseases caused by:
(a) Brucella abortus; (b) Bordetella pertussis; (c) Corynebacterium diphtheriae;
(d) Clostridium tetani; (e) Hemophilus influenzae.
A) (b), (c) and (d)B) (b), (d) and (e)C) (c), (d) and (e).D) (a), (b) and (c)E) (a),
(c) and (e)
B,C,D

174.
The primary function of the dental pulp is to
A) protect the periodontium.
B) provide sensation.
C) assure root-end closure.
D) form dentin.
E) provide nutrition.
D. form dentin

175.

Hepatitis B

1. Anti-HBs and HBs-antigen


2. Anti-HBc and Anti-HBs
3. Anti-HBs no Anti-HBc
window period- have Anti-HBc but not Anti-HBs
o 1. Anti-HBs and HBs-antigen= person is a carrier
o 2. Anti-HBc and Anti-HBs= person was infected, but immune system
fought it off
o 3. Anti-HBs no Anti-HBc= person was immunized window period- have
Anti-HBc but not Anti-HBs

176.
In cases of delayed resorption of primary incisors, the permanent
incisors may be expected to erupt
A) mesial to the normal arch position.
B) distal to the normal arch position.
C) lingual to the normal arch form.
D) facial to the normal arch form
C) lingual

177.
Which of the following teeth is most likely to resist caries?
A) Maxillary lateral incisor
B) Mandibular first premolar
C) Maxillary central incisor

D) Mandibular canine
D) mandibular canine

178.
Based upon morphology, a mesio-occlusal (Class II) cavity preparation
would be most difficult in which of the following teeth?
A) Permanent mandibular first molar
B) Primary mandibular first molar
C) Permanent maxillary second molar
D) Primary mandibular second molar
B because the pulp chambers are higher up into cusp tip and easier to hit the
pulp

179.
The roots of mandibular first premolars are :
(a) flattened faciolingually;
(b) broader facially than lingually;
(c) frequently sharply curved distally;
(d) usually free of marked distal curvature;
(e) frequently seen with slight concave areas on mesial and distal surfaces.
B,D,E

180.
In healing of a fracture, which of the following will prevent favorable
reconstruction and alignment of bone ? (a) Formation of pseudoarthrosis;
(b) Functional remodeling of the procallus by osteoblasts and osteoclasts;
(c) Formation of new bone at the site of fracture;
(d) Presence of sequestrum;
(e) Organization of a hematoma at the site of fracture
(a) and (d)
Sequestrum- a piece of dead bone that has become separated during the process
of necrosis from normal/sound bone.

181.
In a cervical cross section, which premolar(s) sometimes exhibit a root
outline and a pulp chamber floor outline that are both kidney-shaped?

A) All premolars
B) Maxillary second
C) Mandibular second
D) Maxillaryfirst
E) Mandibular first
o D. maxillary 1st premolar
o - has 2 roots
o - kidney shaped bc interradicular groove on the mesial

182.
Ketone Bodies
Synthesis of ketone bodies in liver when E stores are low
- VERY IMPORTANT for brain in starved state bc brain only uses glucose for
E
- in LIVER: free FA's and amino acids metabolized to form ketone bodies
- beta oxidation of alpha-keto acids derived from amino acids
- occurs in mitochondrial matrix of hepatocytes (liver)
- acetyl-coA--> ketone bodies in starved state
Process:
acetyl CoA-- (cholesterol)--(HMG COA synthase= RLS)> acetoacetate-->
beta-hydroxybutyrate (both can be reconverted back to acetyl-CoA and
used in TCA cycle in other tissues)
- 3 ketone bodies: acetoacetate, beta-hydroxybutyrate, acetone (acetone not
used for E bc can't be reconverted in other tissues back to acetyl-CoA to be
used in TCA cycle)--> excreted in urine; causes fruity breath)
- Ketosis= ketone bodies accumulate (get ketoacidosis and diabetic coma)

183.
In cervical cross-section, the root of a mandibular canine is described as
A) broader mesiodistally on the lingual than on the facial.
B) triangular.
C) roughly conical.

D) flattened in a mesiodistal direction.


D) flattened in the mesiodistal direction

184.
Which of the following is characteristic of primary maxillary canines?
A) Cusp is low and rounded.
B) The mesioincisal cusp ridge is shorter than the distoincisal cusp ridge.
C) The mesioincisal cusp ridge is longer than the distoincisal cusp ridge.
D) Crown height < mesiodistal width.
From the facial aspect:
In the permanent max canine, the mesial cusp ridge is shorter than the distal
In the primary max canine, the mesialincisal slope is longer than the
distoincisal
- longer and sharper cusp than permanent max canine

185.
Frequency of impulse activity in the afferent nerve from a muscle
spindle (Group a afferent fibers) is increased by a (an)
(a) relaxation of intrafusal muscle fibers;
(b) increased activity in gammaefferent fibers;
(c) passive stretch of the muscle.
A) (c) onlyB) (b) onlyC) (b) and (c)D) (a) and (c)E) (a) onlyF) (a) and (b)
B and C

A stretch reflex is a muscle contraction in response to stretching within the


muscle. It is a monosynaptic reflex which provides automatic regulation of
skeletal muscle length.
- 1a afferent sensory fiber (proprioception) are parallel to contractile muscle
fibers so they respond to length and stretch
- fusimotor system= muscle spindles and gamma motoneurons--> the system
the CNS uses to control and modify spindle activity; proprioceptive info

- gamma motoneurons regulate gain of stretch reflex by adjusting level of


tension in intrafusal muscle fibers of muscle spindle

186.
Which of the following substances represents an unsaturated fatty acid?
A) CholineB) OleateC) PalmitateD) StearateE) Cholesterol
B) Oleate

187.
Each of the following is involved in gene cloning EXCEPT one. Which
one is this EXCEPTION?
A) Restriction nucleasesB) DNA ligaseC) Reverse transcriptaseD) DNA
polymerase IE) RNA polymerase
E) RNA polymerase

188.
From the incisal aspect, the crown of a maxillary canine normally
exhibits which of the following?
A) A distal portion that displays some concavity in its facial outline
B) A trapezoidal outline
C) Less faciolingual thickness than the crown of a mandibular canine
D) A mesial portion that is thinner faciolingually than the distal portion
E) Symmetry between its mesial and distal portions
o A)
o very prominent distal concavity from distal contact--> CEJ

189.
During swallowing, muscular contraction results in movements that seal
off the oropharynx from the nasopharynx. Which of the following muscles
cause movements that result in a fold in the posterior wall of the pharynx?
A) Tensor veli palatini
B) Musculus Uvulae
C) Levator veli palatini
D) Palatoglossus

E) Palatopharyngeus
E) Palatopharyngeus

190.
Which of the following primary teeth would exhibit a prominent cervical
ridge on both the facial and lingual surfaces?
A) T
B) L
C) A
D) J
E) F
E) F
maxillary central incisor
- no mamelons
- prominent facial and lingual cervical ridges

191.
Which of the following explains why enamel is harder than bone?
A) Enamel contains amelogenins in its organic matrix.
B) Enamel contains more collagen.
C) Enamel crystals are larger and more firmly packed.
D) Enamel contains more magnesium and carbonate.
E) Enamel crystals have more surface area.
C)

192.
in an aqueous solution at pH 7, a peptide containing 1 amino group side
chain and 2 carboxyl group side chains would have which of the following net
charges?
A) 1-B) 2-C) 1+D) 0E) 2+
A) -1
pKa of amino= 9 (+1 in this situation)
pKa of carboxyl group= 4.4 (-2 in this situation)

193.
In which of the following teeth is the mesial portion MOST distinctly
separated from the remainder of the occlusal table by a transverse ridge?
A) Mandibular second premolar
B) Primary maxillary first molar
C) Mandibular first molar
D) Primary mandibular second molar
E) Primary mandibular first molar
o E) primary 1st molar
o - has prominent transverse ridge that connects MB and ML cusps
o - most unique tooth
o - most difficult primary tooth to restore
o - no central fossa; only mesial and distal triangular fossae
o - prominent buccal cervical ridge
o - well developed mesial marginal ridge
o - 4 cusps; MB largest; ML sharpest

194.
On permanent teeth, the greatest incisal curvature of a cervical line is on
which surface of which incisor? (surface --> incisor)
A) Mesial surface --> Maxillary central
B) Mesial surface --> Mandibular lateral
C) Mesial surface --> Mandibular central
D) Distal surface --> Mandibular central
E) Distal surface --> Maxillary lateral
A)
most contoured CEJ on mesial

195.

Which of the following enzymes converts trypsinogen to trypsin?

A) Peptidase B) Secretin C) Pepsin D) Enterokinase


D) Enterokinase/Enteropeptidase
- produced by duodenum wall (Crypts of Lieberkuhn) whenever ingested food
enters duodenum
- converts trypsinogen to active form, indirectly activating of pancreatic
digestive enzymes

196.
From the facial view, the maxillary first molar has its lingual root apex in
line with which of the following?A) Mesiofacial cusp tip
B) Mesiodistal diameter midpoint
C) Distofacial line angle
D) Distofacial cusp tip
E) Facial groove
E) facial groove

197.
Peripheral (lower motor neuron) lesions of cranial nerve VII will cause
which of the following?
A) An ipsilateral flaccid paralysis of the facial musculature
B) An ipsilateral flaccid paralysis of only the lower facial muscles
C) A contralateral paralysis of the facial musculature
D) A contralateral paralysis of only the lower facial muscles
E) A contralateral paralysis of only the upper facial muscles
A)
LMN lesion- ipsilateral paralysis of upper and lower face; loss of corneal
reflex= Belly's Palsy
UMN lesion (central lesion to facial motor nucleus from stroke)- contralateral
lower face weakness only
- can still raise eyebrows on affected side bc bilateral innervation to upperface
muscles

198.
Which of the following represents the location of the cell bodies of pain
fibers in the glossopharyngeal nerve?A) Trigeminal (semilunar) ganglion
B) Superior ganglion of cranial nerve IX

C) Spinal nucleus of cranial nerve V


D) Nucleus ambiguous
E) Otic ganglion
- sensory fibers of CN IX are located in the superior ganglion (jugular foramen)
and inferior petrosal ganglion (jugular foramen)

199.
Which of the following maxillary teeth has the largest cervico-occlusal
crown height?
A) First molar
B) First premolar
C) Second molar
D) Second premolar
B

200.

The midroot cross-sectional diagram illustrates the root of which maxillary


molar?
A) Left third
B) Right first
C) Left first
D) Right third
B) right 1st

201.
Which of the following teeth is MOST likely to have a distal coronal
concavity that can pose special problems in matrix placement?
A) Mandibular first premolar
B) Maxillary first molar
C) Mandibular first molar
D) Maxillary first premolar
B)

202.
Calcification of the mandibular third molars generally begins at
A) 8-10 years.
B) 14-16 years.
C) 5-7 years.
D) 3-4 years.
E) 11-13 years.
A) 8-10 yrs

203.
Which of the following nerves is the MOST likely to become injured in
fractures of the mid-humeral shaft?A) Radial
B) Musculocutaneous
C) Ulnar
D) Median
E) Axillary
A) radial, extrinsic extensors of wrists and hands, cutaneous supply to back of
hand
supplies triceps brachii
-originates from the posterior cord of the brachial plexus with roots from C5,
C6, C7, C8 & T1.
- The radial nerve divides into a deep branch (which becomes the posterior
interosseous nerve), and continues as the superficial branch which goes on to
innervate the dorsum (back) of the hand.
- After giving off branches to the long and medial heads of the triceps brachii, it
enters a groove on the humerus, the radial sulcus.

204.
The lymph vessels that drain both dental arches connect directly with
which of the following nodes?
A) Sublingual B) Retropharyngeal C) Superficial cervical D) Submandibular E)
Deep cervical
D) Submandibular

205.
From a proximal view, which of the following describes the crown
outline on a mandibular posterior tooth?A) It is designed to allow for a
minimum amount of chewing effficiency on the coronal surface of the tooth.

B) It is usually rhomboidal and has a design flaw that encourages cusp fracture.
C) It is usually trapezoidal and has a design flaw that encourages cusp fracture.
D) It is designed to protect against root fracture by having the crown structure
serve as root support
B

206.
Alpha-ketoglutarate, oxygen, and ascorbic acid are essential for which of
the following processes?
A) Hydroxylation of proline
B) Activation of procollagen peptidase
C) Oxidative deamination of lysine
D) Incorporation of proline
E) Gamma-Carboxylation of proline
A)

207.
Which of the following teeth have long axes positioned with their root
apices facial and their crowns lingual?
A) Maxillary molars
B) Mandibular molars
C) Mandibular incisors
D) Maxillary premolars
E) Maxillary incisors
B) mandibular Molars

208.
A derivative of vitamin K is the coenzyme for which of the following?
A) Cross-linking of fibrinogen
B) Production of menadiol
C) Esterification of retinol
D) Carboxylation of glutamate side chains
E) Hydrolysis of peptide bonds
D)
Vit K needed to carboxylate glutamate residues to form gammacarboxyglutamate residues--> allows them to bind Ca2+ needed in blood
coagulation (Factors II, VII, IX, X. protein C and S)

209.
Antidiuretic hormone is PRODUCED by cells that reside in which of the
following structures, cells or areas?A) Pars intermedia
B) Acidophils of the pars distalis
C) Pars nervosa
D) Supraoptic nucleus of the hypothalamus
E) Basophils of the pars distalis
D

210.
The pyramids contain
A) upper and lower motor neuron fibers.
B) lower motor neuron fibers only.
C) pain fibers.
D) upper motor neuron fibers only.
E) sensor fibers.
D

211.
Which of the following anterior primary rami form the medial cordof the
brachial plexus?
A) C-6 and C-7
B) T-2 and T-3
C) C-4 and C-5
D) C-5, C-6, and C-7
E) C-8 and T-1
E)

212.
Preganglionic sympathetic fibers to the head have their cell bodies in the
A) superior cervical ganglia.
B) gray rami communicantes of the thoracic region.
C) intermediolateral horns of the thoracic spinal cord.
D) paravertebral ganglia.
E) cervicothoracic ganglia.
C

213.
In the photomicrograph below of a ground section of a tooth viewed by
transmitted light, some dentinal tubules appear black because they
A) have become filled with air during sectioning.
B) contain deeply stained odontoblastic processes.

C) are filled with minerals.


D) are filled with bacteria.
E) are filled with blood.
A

214.
Fiber tracts passing from the thalamus to the cortex are found in which
of the following?
A) Internal capsule
B) Medial lemniscus
C) Lateral lemniscus
D) Anterior commissure
E) Corpus callosum
A

215.
Which of the following is the MOST numerous cell type in the normal
dental pulp?
A) Odontoblast
B) Macrophage
C) Mast cell
D) Adipocyte
E) Fibroblast
E

216.
Which of the following represents the MOST vascular portion of the
articular disc of the temporomandibular joint?
A) Posterior thick zone (band)
B) Bilaminar zone
C) Anterior thick zone (band)

D) Intermediate thin zone (band)


B

217.
Which of the following organs is the NEAREST to the right kidney?
A) Stomach
B) Colon
C) Pancreas
D) Duodenum
E) Spleen
B

218.
The prochordal plate consists of
A) the cardiogenic mesoderm and the neurectoderm.
B) the cloacal membrane and the overlying amnion.
C) endoderm of the roof of the yolk sac and embryonic ectoderm
.D) embryonic endoderm, mesoderm, and ectoderm.
E) a circular area anterior to the notochord in which the endoderm is fused to
the embryonic mesoderm.
C

219.
Which group of fibers of the periodontal ligament offer the
MOSTresistance to movement of the tooth in an apical direction?
A) Apical
B) Alveolar crest
C) Oblique
D) Interradicular
E) Horizontal
o C) Oblique resists intrusive movements and rotational forces
o

o In the APICAL direction

220.
On a comparative basis, bone and cellular cementum are similar in that
both
A) contain cells in lacunae with canaliculi that extend primarily toward the
nutritional source.
B) are deposited throughout life, acquire blood vessels during aging, and
contain similar collagenous fibers.
C) are difficult to resorb, heal well, and contain cement lines.
D) contain concentric patterns, have Volkmann's canals, and can be cancellous.
A

221.
The core of a microvillus consists of which of the following?
A) Vimentin filaments
B) Microfilaments
C) 9 + 2 arrangement of microtubules
D) 9 triplets of microtubules
E) Intermediate filaments
B

222.
The blood-testis barrier is maintained by
A) myoepithelial cells.
B) spermatogonia.
C) Leydig cells.
D) spermatocytes.
E) Sertoli cells.
E

223.
Oxyphil cells are present in which of the following?
A) Fundic stomach
B) Parathyroid gland
C) Spleen
D) Thymus
E) Neurohypophysis
D

224.
Which of the following is the nerve that supplies the mimetic muscles?
A) Hypoglossal
B) Vagus
C) Spinal accessory
D) Trigeminal
E) Facial
E) mimetic muscles= facial muscles

225.
Which of the following is the posterior limit of the vestibular side of the
cheek?
A) Sulcus terminalis
B) Sphenomandibular ligament
C) Stylomandibular ligament
D) Retromolar pad
E) Pterygomandibular raphe
E

226.
Para-keratinized oral mucosa is often found on which of the following?
A) Attached gingiva
B) Oral surface of the soft palate
C) Ventral surface of the tongue
D) Skin surface of the lips
E) Floor of the mouth
A) attached gingiva

227.
Which of the following represents a popular theory for the force that is
considered to be the MOST responsible for active tooth eruption?
A) Crestal bone formation pulls the tooth.
B) Cells and fibers in the developing PDL pull the tooth toward the oral cavity.
C) Apical fundal bone formation pushes the tooth.
D) Dentin formation in the root pushes the tooth into the oral cavity.
E) Vascular pressure within the pulp pushes the tooth intraorally.
B

228.
Which of the following ligaments helps resist posterior movement of the
mandibular condyles?A) Temporomandibular
B) Anterior cruciate
C) Sphenomandibular
D) Stylomandibular
E) Pterygomandibular raphe
A)

229.
Which of the following structures runs directly adjacent to, and parallel
with, the median nerve in the arm?A) Brachial artery
B) Ulnar artery
C) Radial artery
D) Medial brachial cutaneous nerve
E) Cephalic vein
A

230.
Structure of Thymus
- made up of lobules held together by areolar tissue
- enclosed by capsule
- each follicle has:

1. cortex- mainly lymphoid cells; earliest T cell dev (thymocyte)


2. medulla - Hassall's corpuscle (remains of the epithelial tubes from 3rd
branchial pouch)
- less lymphoid cells

*** Thymus and spleen only have efferent lymphatic vessels


2 origins of thymus:
- endoderm (3rd branchial pouch)
- hematopoeitic stem cells

231.
Which of the following represents the normal substrate of thrombin?
A) Thromboplastin
B) Fibrin
C) Fibrinogen
D) Prothrombin
E) Thrombospondin
C
Substrate= the substance on which an enzyme acts
Prothrombin--> thrombin converts fibrinogen--> fibrin in the clotting cascade

232.
Epinephrine causes an elevation in cAMP levels in muscle cells which in
turn activate
A) glycogen synthetase.
B) ATPase.
C) glycogen phosphorylase.
D) glycogen phosphorylase phosphatase.
E) adenyl cyclase.
C) glycogen phosphorylase

233.
Which of the following noncollagenous protein components
BESTcharacterizes dentin matrix?
A) Osteonectin
B) Laminin
C) Fibronectin
D) Phosphophoryn
E) Vimentin
D)
- one of the proteins formed from dentin sialophosphoprotein

- MOST ACIDIC protein ever discovered


- aa sequence makes it HIGHLY (-)--> attracts A LOT of Ca2+ for dentin
- dentin sialphosphoprotein- only protein produced uniquely by odontoblasts;
non collagenous protein broken into:
1. dentin phosphoprotein
2. dentin sialoprotein
3. dentin glycoprotein

234.
Intravenous injections of KCl solution would increase the secretion of
which of the following?
A) Cortisol
B) Parathyroid hormone
C) Aldosterone
D) Insulin
E) Calcitonin
C) Aldosterone- uptake of Na+ from DCT of nephron

235.
The movement of Ca++ across a membrane is characterized by which of
the following?
A) An example of a cotransport system
B) Maintains Ca++ concentration very much higher in the cell than in the
extracellular fluid
C) A facilitated diffusion
D) In many cell membranes involves a Ca++Na+countertransport system
D)

236.
Which of the following is necessary for de novo synthesis of
cholesterol?
A) NADP+
B) NA+
C) FADH2
D) NADH
E) NADPH
E) NAPH (reduced)
- used in anabolic processes (steroid and FA synthesis) as a supply of reducing

equivalents
RLS in synthesis of cholesterol: HMG CoA Reductase
-converts HMG-CoA--> mevalonate
- 2/3 plasma cholesterol is esterified by LCAT (lecithin-cholesterol
acyltransferase)

237.
The enzyme catalyzing the rate-controlling step in the de novosynthesis
of fatty acids is regulated allosterically by the positive modulator
A) cyclic AMP.
B) ATP.
C) oxaloacetate
.D) citrate.
E) NADPH.
D) citrate
- citrate builds up in TCA when have a high E state- want to make FAs when
have excess E
- citrate also inhibits glycolysis

238.
Which of the following controls the excitability of the muscle spindle?
A) Gamma efferent system
B) Alpha efferent discharge
C) Length of the extrafusal fibers
D) Load on the muscle
A) gammer efferent
- stretch reflex
- muscle spindle= sensory receptor in belly of extrafusal fibers (muscle)
- length of muscle
- afferent Ia and II
- 3 types of intrafusal fibers:
1. nuclear bag (length and tension)- dynamic fast type Ia
2. nuclear bag- static
3. nuclear chain fibers (length and tension)- slow type II afferent

- muscle spindles receive efferent= gamma innervation--> causes shortening of


muscle spindle

239.
Under strict anaerobic conditions, the catabolism of one glucose
molecule would yield a net of
A) 4 ATP and 2 pyruvic acid molecules.
B) 2 ATP and 2 lactic acid molecules.
C) 4 ATP and 2 lactic acid molecules.
D) 2 ATP and 2 pyruvic acid molecules.
B)
ANAEROBIC- Lactic fermentation:
- pyruvate converted to lactic acid via LACTATE DEHYDROGENASE
- need to convert NADH--> NAD+ so glycolysis can continue
- MUSCLE TISSUE (working out) and some BACTERIA

240.
Which of the following represents a polyunsaturated fatty acid that is
commonly found in animal cell membranes?
A) Linoleic
B) Stearic
C) Oleic
D) Sialic
E) Lactic
A)
#C's Common Name
12Lauric acid
14Myristic acid
16Palmitic acid
Unsaturated- drop in MP
18Stearic acid
18 Oleic acid
18Linoleic acid
18alpha-linolenic acid

241.
GABA increases the permeability of postsynaptic membranes to which
of the following ions?
A) Potassium
B) Sodium
C) Magnesium
D) Calcium
E) Chloride
o E) Clo - most abundant inhibitory NT

242.

Formation of Microfilaments
o - G-actin= globular actin (individual subunits of actin)
o - F-actin= G-actin subunits assemble into filamentous polymers
o - 2 parallel f-actin strands make up a microfilament (of cytoskeleton)
o - requires ATP to transform G-actin--> f-actin

243.
Which of the following sweeteners is non-nutritive as well as noncariogenic?
A) Maltose
B) Galactose
C) Sorbitol
D) D-fructose
E) Saccharin
E)
- artificial sweetener
- gives no food E

244.
If the pH becomes lower than the isoelectric point of a protein, then how
will the protein respond in an electrophoretic system? It will

A) migrate to the positive pole


.B) separate into its different monomeric forms.
C) remain stationary and unchanged.
D) migrate to the negative pole.
E) become denatured.
D)
IP= pH at which the protein is neutral

245.
Sodium fluoride inhibits glycolysis by affecting which of the following?
A) Phosphatase
B) Amylase
C) Phosphorylase
D) Enolase
D) enolase

246.
Insulin INCREASES the activity of which of the following?
A) Phosphofructokinase
B) Glucose-6-phosphatase
C) Phosphorylase a
D) Fructose 1-6 diphosphatase
E) Enolase
A
- high E state, insulin stimulates synthesis of ATP from glucose

247.
When arterial pressure increases, pressoreceptors discharge and
A) cause vasoconstriction throughout the peripheral circulatory system.
B) excite sympathetic nerves and inhibit parasympathetic nerves.
C) inhibit tonic activity of sympathetic nerves and excite parasympathetic
nerves.
D) increase cardiac rate and strength of contraction.
C

248.
The level of nonprotein nitrogen in the blood is due principally to the
level of which of the following?A) Arginine
B) Creatine
C) Uric acid
D) Ammonia
E) Urea
After prolonged acidosis, which of the following represents the nitrogenous
product that is excreted in high amounts in the urine?
A) Aspartic acid
B) Creatinine
C) Urea
D) Uric acid
E) Ammonia
o E) Urea
o E) ammonia

249.
Which of the following are the immediate effects of calcitonin on serum
levels of calcium and phosphate?A) Serum calcium decreases, Serum
phosphate has no change
B) Serum calcium has no change, Serum phosphate increases
C) Serum calcium increases, Serum phosphate increases
D) Serum calcium decreases, Serum phosphate decreases
E) Serum calcium increases, Serum phosphate decreases
D) calcium and phosphate are put back into bone

250.
Most endogenous cholesterol in the liver is usually converted into which
of the following?
A) Cholic acid
B) Glucose
C) Oxaloacetate
D) Ketone bodies
E) Steroids
A) Cholic acid
cholic acid= bile acid

- produced by liver from cholesterol


- derivatives are made from cholyl-CoA which forms a conjugate with either
glycine, or taurine, yielding glycocholic and taurocholic acid respectively.
- Cholic acid and chenodeoxycholic acid are the most important human bile
acids

251.
The ability to concentrate urine varies among animal species. The
maximum urine concentration that can be produced by an animal isMOST
closely related to which of the following?
A) Total number of nephrons
B) Diameter of the distal tubule
C) Length of the loop of Henle
D) Renal blood flow
E) Glomerular filtration rate
C)

252.
In relative insulin insufficiency, acetyl CoA is usually channeled into
A) gluconeogenesis.
B) the Krebs cycle.
C) ketone-body formation.
D) cholesterol synthesis.
E) fatty-acid synthesis.
o C)
o - insulin insufficiency indicates a post-absorptive state (low serum
glucose)
o - Diabetes pts- low insulin--> get diabetic ketoacidosis bc form ketone
bodies for E

253.
The capsule of Streptococcus mutans is an important virulence factor
that
A) causes circulatory collapse.
B) is necessary to cause bacteremia.
C) prevents phagocytic digestion.

D) enhances oral accumulation.


E) has proteolytic activity.
D)

254.
Which of the following disorders is LEAST likely to be included in the
differential diagnosis of a patient with acute appendicitis?
A) Pelvic inflammatory disease
B) Crohn's disease
C) Duodenal peptic ulcer
D) Gastroenteritis with mesenteric adenitis
E) Meckel's diverticulitis
C

255.
Uncomplicated healing of a wound by secondary intention, observed
microscopically at three days, is MOST likely to show evidence of which of the
following?
A) Lack of acute inflammation
B) Ulceration of the epithelial surface
C) Keloid formation
D) Granulomatous inflammation
E) Mature cicatrix
B)?
Classic model of wound healing:
1. hemostasis
2. inflammatory- bacteria/debris phagocytized and removed
1. PMNs- 1st 2 days + T cells--> cleanse wound
2. Macrophages- after 2 days
- essential to wound healing (phagocytize bacteria and damaged tissue)
3. proliferative
- angiogenesis by vascular endothelial cells
- collagen deposition by fibroblasts
- granulation tissue formation (type III)--> replaced later with type I
- epithelialization- epithelial cells proliferate and crawl up wound bed to cover
- wound contraction by myofibroblasts
4. remodeling - collagen remodeled and realigned along tensions lines; cells no
longer needed undergo apoptosis

1. Primary intention- e.g. surgical incision


- no/minimal scar formation
- wound edges line up
2. Secondary intention- e.g. gingivectomy, gingivoplasty, tooth extraction
- granulates--> scar formation- slower healing + wound care
3. Tertiary- e.g. healing of wounds by use of tissue grafts
- wound is cleaned and observed; left open

256.
Patients with which of the following malignancies have the poorest
prognosis?
A) Pancreatic carcinoma
B) Squamous carcinoma of the tongue
C) Malignant melanoma
D) Lung cancer
E) Carcinoma of the colon
A)
Pancreatic cancer
- poor prognosis
- 3 yr survival rate is 30%
- "silent killer" bc usually doesn't cause symptoms
- later symptoms are non-specific: jaundice, pain in upper abdomen, weight
loss
Risk factors:
- male
- age
- smoking
- obesity
- diabetes mellitis
- H. pylori
- family history
- gingivitis or perio disease

257.
Which of the following is the MOST important function of bacterial pili
in causing human infectious disease?A) Allowing bacteria to adhere to human
cells
B) Destroying phagocytic macrophages and neutrophils

C) Making the bacteria motile


D) Transfering DNA from one bacterium to another
A)
fimbriae= short pilus used to attach bacteria to host surface
- mutant bacteria that lack fimbriae can't attach to surface and can't cause
disease
- found in gram - and gram + bacteria

258.
A patient with rheumatic heart disease is MOST likely to develop
congestive heart failure due which of the following?
A) Cor pulmonale
B) Heart murmur
C) Valvular insufficiency
D) Aschoff bodies in coronary arteries
E) Digitalis toxicity
C)

259.
Which of the following are directly associated with destruction of
glomerular basement membranes in a patient with glomerulonephritis?
A) Polymorphonuclear leukocytes
B) IgE antibodies
C) Eosinophils
D) Lymphokines
A)

260.
If Type A blood is accidentally transfused into a Type B recipient, the
immediate hemolytic reaction would be the result of which of the following?
A) T cytotoxic cell activation
B) IgG against the B antigen
C) IgM against the A antigen
D) Sensitization to antigens other than A or B
E) IgA against the A antigen
C

261.
A patient who has anemia, poorly localized abdominal pain, and wrist
and foot drop probably is manifesting a toxic state induced by which of the
following?
A) Mercury
B) Carbon tetrachloride
C) Lead
D) Bismuth
E) Carbon monoxide
D) Lead
- pain, muscle weakness, paresthesia, NVD, diarrhea, constipation, poor
appetite
- black line on gingiva
Bismuth
- one of the least toxic of the heavy metals
- mostly kidney and liver
- skin and respiratory irritation
- "bismuth line"= black line on gingiva
Mercury
- peripheral neuropathy- paresthesia, itching, burning pain
- skin discoloration- pink cheeks, fingertips toes
- blocks breakdown of catecholamines- -> sweating, tachycardia, salivation,
HTN
- kidney dysfunction, hypotonia of muscle
CCl4
- CNS
- degenerate liver and kidneys--> can lead to cancer
- prolonged--> coma and death

262.
Which of the following is the major VIRAL cause of birth defects in
infants in developed countries?A) Varicella-zoster
B) Rubella
C) Measles
D) Herpes simplex
E) Cytomegalovirus
E) CMV

TORCHeS

263.
Phage conversion is responsible for which o'f the following?
A) Production of pyrogenic toxin
B) Transformation of bacteria
C) Ability of Rhizobium species to fix nitrogen
D) Production of beta-galactose
A)
Bacteriophage= vector for transduction from one cell to another

264.
MOST rickettsial diseases can produce severe illness in humans because
rickettsiae
A) are destructive for endothelial cells.
B) are destructive for epithelial cells.
C) produce potent exotoxins.
D) cause extensive CNS damage.
A)

265.
Hereditary angioedema is the result of which the following?
A) Deficiency in C1 esterase inhibitor
B) Defective VH gene recombination to DJH
C) Lack of a thymus
D) Bare Iymphocytes (no Class I/II antigens)
E) Developmental arrest of Iymphocytes
A)
- presents in 20's-40s
- local swelling in subcutaneous tissues
- doesn't respond to antihistamines, corticosteroids, or Epi
Cause:
- deficiency of C1 esterase inhibitor
- C1 inhibitor inhibits C1, Factor XII, and kallikrein--> therefore give rise to

vasoactive substances when unregulated


Tx
- want to treat the edema, esp if larynx
- AUTOSOMAL DOMINANT

266.
Which of the following viral diseases has the LONGESTincubation
period?
A) Measles
B) Influenza
C) Herpetic gingivostomatitis
D) Rabies
E) Common cold
D)
Rabies
- viral disease
- causes acute encephalitis
- travels to brain following peripheral nerves
- zoonotic- bite from infected animal
- incubation period is few months in humans--> once reach CNS, symptoms
show and fatal within 7 days
- mania, lethargy, depression, uncontroled excitement

267.
Adult respiratory distress syndrome might be caused by each of the
following EXCEPT one. Which one is this EXCEPTION?
A) Heroin overdose
B) Breathing 100 percent O2
C) Shock
D) Viral pneumonia
E) Cigarette smoking
E)
ARDS "Acute" is more appropriate
- serious rxn to various injuries of lung
- severe lung disease

- inflamm of lung parenchmya= impaired gas exchange--> inflamm,


hypoxemia, multiple organ failure
- often fatal and need to be put in ICU on mechanical ventilation
- most commonly associated w/ diffuse alveolar damage

268.
Normal clotting time and normal platelet count accompanied by
prolonged bleeding time suggest which of the following?
A) Thrombocytopenic purpura
B) Excessive aspirin ingestion
C) Hemophilia
D) Steatorrhea
E) Polycythemia vera
B)
Aspirin inhibits the COX pathway--> prostaglandins (PCI2 and TXA2)
- Thromboxane A2 in charge of platelet aggregation

269.
Which of the following bacterial vaccines is given routinely to children
in the U.S.A.?
A) Measles, mumps, rubella
B) BCG
C) Cholera
D) Diptheria, pertussis, tetanus
E) Polyvalent pneumococcus
D
DPT vaccine
- diphtheria and tetanus toxoids
- killed whole cells of bordetella pertussis
- 5 doses btwn 2 months and 5 yrs

270.
Normal human cells contain gene sequences homologous to virus
genome sequences known to induce cancer in animals. What term applies to
these gene sequences when found in human cells?
A) Analogous chromosomes

B) Retroviral genes
C) Protooncogenes
D) Homologous chromosomes
E) Viral oncogenes
C)

271.
If a foreign antigen enters the body through the skin, which of the
following portions of the lymphoid system is the antigen MOSTlikely to reach
FIRST?
A) Lymph nodes
B) Thymus
C) Liver
D) Mucosal-associated lymphoid tissue
E) Spleen
A)

272.
Which of the following statements is CORRECT regarding glioblastoma
multiforme?
A) Its prognosis is generally more favorable than Grade I astrocytoma.
B) The tumor is most common before puberty.
C) It is classified as a type of meningioma.
D) It is derived from the epithelial lining of the ventricles.
E) It is the most common type of astrocytoma.
E)
- it is a type IV astrocytoma
- most common primary brain tumor/astrocytoma
- butterfly glioma bc crosses corpus callosum
- v. poor prognosis <1 yr survival
- found in cerebral hemispheres

273.
The appearance of new strains of human immunodeficiency virus are
primarily the result of
A) errors in translation.
B) natural selection.
C) errors in transcription.

D) genomic recombination
C

274.
Each of the following is commonly associated with congestive heart
failure EXCEPT one. Which one is this EXCEPTION?
A) Anasarca
B) Ankle edema
C) Passive congestion of the liver
D) Dyspnea
E) Cyanosis
A)
"anasarca"= extreme generalized edema
- widespread swelling of skin due to effusion of fluid into EC space
- usually caused by renal, liver, heart failure

275.
Rifampin (Rifadin) is effective in treating active tuberculosis because
it targets which of the following pathways?
A) Translation
B) mRNA splicing
C) Replication
D) Mitosis
E) Transcription
E) transcription

Rifampin
- bactericidal
- administered several months w/out a break w/ other drugs to prevent
resistance
- used along w/ isoniazid, ethambutol, pyrazinamide, streptomycin

276.
A patient bites down rapidly on an unexpected hard surface while
chewing. Cessation of motor unit recruitment in jaw closing muscles is caused
by stimulation of
A) periodontal mechanoreceptors.
B) muscle spindles.
C) nociceptors in the dental pulp.
D) mucosal mechanoreceptors.
A) periodontal mechanoreceptors

277.
In hydrocephalus, excess cerebral spinal fluid is found within which of
the following?
A) Subarachnoid space
B) Ventricle
C) Subdural space
D) Superior sagittal sinus
E) Cisterna magna
B) ventricle
- aka "water on the brain"
- abnormal accumulation of CSF in ventricles of brain
- increases intracranial pressure in skull
- enlarged head
- convulsion/seizure
- mental disability

278.
Which of the following primary grooves uniting in the distal pit on the
occlusal surface of the mandibular second molar represents the one that

normally has no counterpart in the distal pit of the first molar?


A) Distal marginal
B) DL triangular
C) Distal portion of the central
D) DF triangular
E) Distolingual
D) DF Triangular groove
bc of the presence of the small distal cusp on the mandibular 1st molar

279.
Which of the following conditions represents an intoxication rather than
an infection?
A) Salmonellosis septicemia
B) Chancroid
C) Bacteroidosis
D) Anthrax
E) Botulism
E) Botulism
aka botulinis intoxication
- paralytic disease caused by botulinum toxin from C. botulinum (gram +
anaerobic spore forming rod)
- bacteria colonizes the GI tract, enters from wounds, or from food (esp canned
food)
- babies- "floppy baby syndrome"
- toxin is killed by heat
- paralysis usually starts at face and spread to limbs; can get respiratory failure
- toxin inhibits Ach release at NMJ

280.
Which of the following best describes the cervical margin on the facial
crown surface of the maxillary first molar?
A) Irregularly convex toward the apex
B) Evenly convex toward the occlusal
C) Irregularly convex toward the occlusal
D) Evenly convex toward the apexE) Straight
A) Irregularly convex toward apex

281.
Which of the following teeth represents the one most likely to present
with three roots?
A) Maxillary second premolar
B) Mandibular central incisor
C) Maxillary first premolar
D) Mandibular second premolar
E) Mandibular canine
A) Maxillary 1st premolar
- this tooth normally has 2 roots (facial and lingual)
- kidney shaped bc of the interradicular groove

282.
A patient has an extremely wide, notched tooth in the mandibular left
central incisor position. Clinical and radiographic examinations reveal 28 teeth
have erupted, but four third molars have not erupted. Which of the following
conditions exists?
A) Gemination
B) Dilaceration
C) Fusion
D) Concrescence
E) Dens in dente
A) Gemination- 2 crowns dev from 1 tooth bud
- share a single root and root canal--> wide, notched tooth
c) fusion- 2 tooth buds fuse; dentin is connected
d) concrescence- 2 completely separate teeth fused at cementum

283.

Steroid-induced osteoporosis mechanisms of action:


o 1. directly inhibit osteoblasts
o 2. directly enhance bone resorption
o 3. inhibit GI Ca2+ absorption
o 4. increase urine Ca2+ loss
o 5. inhibit sex steroids

284.
Most common location for atherosclerotic aneurysm?
abdominal aorta> coronary artery

285.
Initially, the developing heart is
A) positioned anterior to the prochordal plate.
B) between the prochordal plate and the notochord.
C) positioned potsterior to the notochord.
D) induced by the notochord.
A

286.
Cells that will form the vertebrae have their origin in which of the
following?
A) Two pairs of somites
B) Notochord
C) Neural arch
D) Intermediate mesodermal plate
E) Dermamyotome
A)
Somite
- present in developing embryo of vertebrates
- somite= masses of mesoderm along 2 sides of neural tube
- eventually becomes:
1. vertebrae (sclerotome) and rib cartilage
2. skeletal muscle (myotome)
3. dermis (dermotome)

- specify migration paths of neural crest cells and spinal nerve axons

287.
A 5-year-old child with vesicular lesions limited to the palate and the
posterior oropharyngeal mucosa has an oral temperature of 101F (38C). The
most probable diagnosis is
A) recurrent oral herpes.
B) hand-foot-and-mouth disease.
C) herpangina.
D) chickenpox
.E) herpetic gingivostomatitis.
C) herpangina- mostly in children
- painful ulcers/vesicles in posterior oral cavity and pharynx- tonsillar pillars,
soft palate
- self-limited <1 week
- Coxsackie A virus
b) hand-foot-mouth
- usually children; fecal-oral
- anywhere in oral cavity; esp palate, tongue, buccal mucosa
a) recurrent oral herpes- most occur at vermillion of lips "herpes labialis",
HARD PALATE

-recurrent is usually UNILATERAL


- HSV1 stays latent in trigeminal ganglion

288.
When the mandible performs a laterotrusive movement, the
laterotrusive-side condyle moves primarily about which of the following axes?
A) Sagittal
B) Horizontal
C) Transverse
D) Vertical
D) vertical
- moves down and laterally

289.
When viewed from the sagittal plane, the axial inclination of the anterior
teeth
A) inclines distally.
B) inclines mesially.
C) inclines lingually.
D) inclines facially.
E) remains vertical.
D)

290.
Each of the following is attributable to hepatic failure EXCEPT one.
Which one is the EXCEPTION?
A) Spider telangectasia
B) Hypoalbuminemia
C) Gynecomastia
D) Tremor
E) Mallory bodies
o E) Mallory bodies

o
o - aka alcoholic hyaline
o - inclusion found in liver cells of people suffering from alcoholic liver
disease
o - pink, eosinophilic
o - characteristic twisted rope appearance
o - most common in ALCOHOLIC HEPATITIS and ALCOHOLIC
CIRRHOSIS

291.
Between which of the following permanent teeth is the lingual
embrasure smaller than the facial embrasure?
A) Mandibular first molar and mandibular second molar
B) Maxillary first premolar and maxillary second premolar
C) Mandibular first premolar and mandibular second premolar
D) Maxillary second molar and maxillary third molar
C)

292.
Protein and RNA synthesis occur in each of the following phases of the
cell cycle EXCEPT one. Which one is the EXCEPTION?
A) M
B) G2
C) G1
D) G0
E) S
A)
- mitosis involves division

293.
A patient presents with symptoms of an acute abscess on the maxillary
left lateral incisor. There are no clinical signs of decay or restoration.
Radiographically, which of the following will most likely be shown as the
cause of the abscess?
A) Dilaceration
B) Concrescence
C) Enamel pearls
D) Agenesis
E) Dens in dente
E) Dens in dente
- outer surface of tooth folds inward
- coronal and radicular forms
- from infolding of dental papilla during tooth dev
- most common in MAX LATERAL INCISORS
- the malformation frequently results in early pulp NECROSIS

294.
In general, enzyme-deficiency diseases are inherited by which of the
following modes?
A) Autosomal recessive
B) Polygenic
C) Autosomal dominant
D) X-linked dominant
A)
X-linked:

1. Fabry's- lyososome storage disease


- lack alpha-galactosidase; build up ceramides
- FABR
- failure of heart
- angiokeratomas
- brain problems
- renal failure
2. Hunter's- gargoylism, mental retardation, no corneal clouding (unlike
hurler's)
- x-linked recessive
- Mucopolysaccharide storage disease
- deficient in L-iduronate sulfatase
- accumulation of heparan sulfate, dermatan sulfate

295.

The bulk of a tooth consists of

A) dentin.
B) cementum.
C) enamel.
D) crown.
E) pulp.
A) Dentin

296.
Nongonococcal urethritis is often caused by microorganisms of which of
the following genera?
A) Treponema
B) Hemophilus
C) Chlamydia
D) Neisseria
C) Chlamydia Trachomatis (Chalmys= cloak)
- obligate intracellular organisms that cause mucosal infections; can't make own
ATP
1. Arthritis
2. nongonococcal urethritis
3. conjunctivitis
4. PID
1. Types A,B,C- chronic infection; blindness in Africa

2. Type D,K- urethritis, PID, ectopic pregnancy, neonatal pneumonia, neonatal


conjunctivitis
3. Types L1, L2, L3- lymphogranuloma venereum (acute lymphadenitis)
Tx- oral erythromycin or tetracycline

297.
Which of the following is least likely to contribute to or affect stability
of the dental arch form?
A) Interproximal contact form
B) Occlusal contact forces
C) Forces exerted by the lips and tongue
D) Plane of occlusion
E) Periodontal health
D) plane of occlusion

298.
Each of the following grooves originates in the central pit of the
maxillary second molar (four cusp type) EXCEPTION one. Which one is the
EXCEPTION?
A) Distolingual
B) Buccal
C) Transverse grooves of the oblique ridge.
D) Central
o A) distolingual groove
o - from distal pit

299.
Mucosa of the anterior two-thirds of the tongue develops primarily from
A) lateral lingual swellings.
B) hypobranchial eminence.
C) tuberculum impar.
D) Rathke's pouch.
E) foregut endoderm.
A) Lateral lingual swellings

- 3rd week of embryological dev


- lateral lingual swellings dev from mandibular arch; meet in the middle line-->
form median sulcus of tongue post-embryologically
- tuberculum impar- part anterior to foramen cecum
- Rathke's pouch (oral ectoderm)- depression in roof of developing mouth-->
gives rise to anterior pituitary

300.
Which of the following muscles participates in flexion at the glenohumeral and the humero-ulnar joints?A) Brachialis
B) Triceps brachii
C) Deltoid
D) Coracobrachialis
E) Biceps brachii
E) Biceps brachii
- rotates forearm and flexes elbow
1. short head attaches to coracoid process of scapula
2. long head attaches to supraglenoid tubercle of scapula in joint capsule
--> oth insert into radial tuberosity

301.
When viewed from the frontal plane and progressing posteriorly, the
axial inclination of the crowns of maxillary posterior teeth
A) inclines mesially.
B) inclines distally.
C) remains vertical.
D) inclines buccally.
E) inclines lingually.
D) inclines buccally
- maxillary teeth have a lingual/buccal axial inclination
- mandibular teeth have a lingual inclination

302.
Submucosal glands are usually located in the
A) duodenum.
B) jejunum.
C) colon.
D) appendix.
E) fundus of stomach.
A) duodenum

303.
Which of the following terms refers to the presence of digested blood in
the stool?
A) Icterus
B) Hemosiderosis
C) Melena
D) Hematoma
E) Hemochromatosis
D) Melena
- black feces associated w/ GI hemorrhage
- oxidation of Fe when pass through ileum and colon
- most common cause is PEPTIC ULCER DISEASE (H. pylori)
a) icterus= jaundice
e) hemochromatosis- iron overload; deposit in tissue as hemosiderin
(hemosiderosis)
- organs most commonly affected: LIVER, HEART, ENDOCRINE GLANDS
- cirrhosis
- diabetes- pancreatic islet failure
- arthritis- iron deposits in joints
- cardiomyopathy
-testicular failure
- tanning
HEMOCHROMATOSIS IS "HEL"

304.
Soft and hard tissue necrosis characterizes which of the following fungal
diseases?
A) Cryptococcosis
B) Histoplasmosis
C) Candidiasis

D) Mucormycosis
E) Coccidioidomycosis
D) Mucormycosis- Mucor and Rhizopus
- opportunistic fungal infection
- MOLD
- nonseptate hyphae
- disease mostly in KETOACIDOTIC DIABETIC AND LEUKEMIC pts
- proliferate in walls of blood vessels and cause infarction and NECROSIS;
frontal lobe abscesses

*** Coccidoidomycosis (meningitis) and Histoplasmosis are SYSTEMIC


MYCOSIS --> PNEUMONIA AND TB-like granulomas

305.
When in its normal position relative to the arch form, the crown of a
mandibular first molar inclines
A) mesially and facially.
B) distally and lingually.
C) distally and facially.
D) mesially and lingually.
D)
roots incline distally

306.
Which of the following statements about norepinephrine is correct?
A) Is the preganglionic sympathetic neurotransmitter
B) Causes cardiac acceleration
C) Causes general vasodilation
D) Causes vasodilation in vessels of the skin
E) Has a negative inotropic effect on the heart
B

307.
The characteristic of the aorta that is most responsible for the
maintenance of diastolic blood pressure is its

A) proximity to the heart.


B) great peripheral resistance.
C) elastic distensibility.
D) active contraction
E) wide lumen.
C

308.
In the upper limb, which of the following represents a hallmark of
lymphatic vessels?
A) Follow the veins
B) Always travel in pairs
C) Only found on the anterior surface of the limb
D) Contain fenestrations to allow passage of fluids into the interstitium
E) Contain valves
A) Follow the veins

309.
Which of the following characterizes both active transport and facilitated
diffusion?
A) Competitive inhibition
B) Transport bidirectional
C) Transport against a concentration gradient
D) Hydrolysis of ATP
A

310.
Red hepatization refers to which of the following?
A) Congestion of the liver caused by chronic right sided heart failure
B) Stage of lobar pneumonia
C) Late stage of acute hepatitis
D) Cirrhosis caused by hemochromatosis
E) Early stage of acute hepatitis
B) Stage of lobar pneumonia
pneumonia= abnormal inflammation of lung
- Lobar pneumonia as acute progression:
1. congestion 1st 24 hrs

2. red hepatisation or consolidation


3. gray " "
4. resolution/recovery
*** Most common organism that causes lobar pneumonia=
STREPTOCOCCUS PNEUMONIAE "pneumococcus"

311.
Which groove of the mandibular first molar does the maxillary
mesiolingual cusp pass through in a lateral excursive movement on the working
side?
A) Lingual
B) Distobuccal
C) Buccal
D) Central
A

312.
Which of the following types of blotting can be used to identify DNA
restriction fragments?
A) Northern
B) Western
C) Southern
D) Eastern
C) Southern
SNoW DRoP
Southern= DNA
Northern= RNA
Western= protein

313.
Fibrocartilage normally occurs in
A) external ear.
B) C-shaped rings in the wall of the trachea.
C) intervertebral discs.
D) epiglottis.

E) epiphyseal plate.
C) Intervertebral discs
- also in articular cartilage of condyle
a) elastic cartilage
b) hyaline
d) elastic cartilage
e) hyaline

314.
The neurons of the central nervous system that innervate muscles
derived from branchial arches are found in which of the following nuclei?
A) Dorsal motor nucleus of X
B) Nucleus ambiguus
C) Nucleus of Edinger-Westphal
D) Superior salivatory nucleus
E) Hypoglossal nucleus
B) nucleus ambiguous
a) dorsal motor nucleus of X
- general visceral efferent GVE
- secretomotor
- efferent: parasympathetics to viscera of neck, thorax, abdominal cavities to L
colic flexure
c) nucleus of edinger-westphal= EYES
- CN III oculomotor
- GVE: misosis, convergence, accomodation
d) superior salivatory nucleus
- CN VII
- GVE secretomotor: submandibular, sublingual glands (submandibular
ganglion)
- lacrimal glands and sinuses (PP ganglion)

315.
Multiple, lytic lesions of bone characterize each of the following
conditions EXCEPT one. Which is the EXCEPTION?
A) Metastatic carcinoma

B) Osteogenesis imperfecta
C) Langerhans (eosinophilic) granulomatosis
D) Multiple myeloma
E) Hyperparathyroidism
B)

316.
The human immunodeficiency virus preferentially infects which of the
following cells?
A) Cytotoxic T
B) NK
C) Suppressor T
D) Helper T
Each of the following fluids is considered one that can transmit HIV EXCEPT
one. Which one is the EXCEPTION?
A) Breast milk
B) Saliva
C) Amniotic fluid
D) Semen
E) Serum
D) Helper T
B) Saliva

317.
If the anticodon on transfer-RNA is 5'ACG3', then which of the
following is its corresponding codon on messenger-RNA?
A) 5' TGC 3'
B) 5' CGT 3'
C) 5' UAG 3'
D) 5' UGC 3'
E) 5' CGU 3'
o E)
o ALWAYS WRITE IN A 5'--> 3' direction!

318.
Which main product of protein nitrogen metabolism is found in human
urine?
A) Ammonia
B) Creatine
C) Creatinine
D) Urea
E) Uric acid
D) Urea
UREA CYCLE in liver:
- amino groups from AMMONIA and L-ASPARTATE--> urea
- urea is found in blood; excreted as urine; a little as sweat

319.
Which of the following represents the location of the lingual height of
contour on the crown of the mandibular second premolar?
A) Same third as the lingual height of contour on the crown of the mandibular
first premolar
B) Middle third
C) Occlusal third
D) Same third as that tooth's buccal height of contour
E) Same third as the lingual height of contour on the crown of the maxillary
premolars
C

320.
Which of the following cells are the most radiosensitive?
A) Epithelial cells
B) Chondrocytes
C) Fibroblasts
D) Neurons
E) Lymphocytes
E) lymphocytes
- the least differentiated the cell, the more susceptible it is to radiation

321.
Each of the following statements about poliovirus infections is correct
EXCEPT one. Which one is the EXCEPTION?
A) Virus is predominantly shed from the body and transmitted in respiratory
secretions.
B) Some damaged neurons may be repaired, restoring lost functions.
C) Paralysis is an uncommon outcome of infection.
D) There are 3 types of poliovirus, making 3 infections possible.
E) Most infections are subclinical.
A) "polio= gray--> spinal cord" + "itis"= inflammation
Picornavirus + strand RNA
- 3 diff types w/ diff capsid protein
- human enterovirus--> GI tract
- spread via fecal-oral route; highly contagious human-to-human contact
- 90% are asymptomatic
- if enter bloodstream--> can enter CNS= muscle weakness and flaccid
paralysis
- Poliomyelitis "polio" or "infantile paralysis"

322.
In the temporomandibular joint, a very dense collection of organized
elastic fibers is found in which of the following areas of the articular disc?
A) Posterior-superior lamina of the bilaminar zone
B) Posterior-inferior lamina of the bilaminar zone
C) Posterior band
D) Anterior band
E) Intermediate band
A

323.
Which of the following represents the mechanism of action of diphtheria
toxin?
A) Inhibits transcription
B) Inhibits DNA replication
C) Activates cAMP
D) Causes cytolysis
E) Inhibits translation
D

Corynebacterium diPHtheriae
- causes PHARYNGITIS and PSEUDOMEMBRANE in throat
MOA- inhibit elongation factor 2

324.
A stab wound creating a pneumothorax on the left side will usually result
in collapse of which of the following?
A) The left lung and pericardial sac
B) Both lungs
C) The left lung only
D) The right lung only
E) The rib cage on the left side
C
Tension pneumothorax- life threatening condition that results from progressing
of a simple pneumothorax--> forms one-way valve at the point of rupture so air
is trapped and P is put on the lung (can't inflate completely)
- trachea moves to the unaffected side
- similar to cardiac tamponade--> differentiate the 2 w/ x-ray
Most common causes of pneumothoroax:
1. spontaneously (males w/ marfan syndrome)
2. stab wound

325.
Plasmid-mediated antibiotic resistance has been observed in diseases
caused by each of the following EXCEPT one. Which one is the EXCEPTION?
A) Haemophilus influenzae
B) Streptococcus pyogenes
C) Staphylococcus aureus
D) Neisseria gonorrhoeae
E) Bordetella pertussis
B

326.
Which of the following conditions is most commonly associated with
acute pancreatitis?

A) Viral infection
B) Physical trauma
C) Chronic alcohol abuse
D) Hypercalcemia
E) Diabetes mellitus
C
- autodigestion of pancreas by pancreatic enzymes
-I GET SMASHeD (causes):
Idiopathic
IN ORDER OF MOST FREQUENT CAUSES:
1. gallstones
2. ethanol
3. trauma
4. steroids
5. mumps
6. autoimmune disease
7. scorpion sting/snake bite
8. hypercalcemia/hyperlipidemia/hypothermia
9. Drugs (e.g. sulfa)
Clinical presentation:
- epigastric abdominal pain radiating to back
- anorexia
- nausea
*** ELEVATED LIPASE
Complications:
- can lead to DIC
- ARDS (pancreatic enzymes ac on lungs)
- diffuse fat necrosis (lipase)
- pseudocyst formation
- hemorrhage
- infectio
- hypocalcemia (Ca2_ collects in panreatic soap deposits
Chronic Pancreatitis- associated w/ ALCOHOLISM
Acute- #1 cause= GALLSTONES

327.
Ameloblasts will not differentiate from preameloblasts unless they
A) contact neural crest mesoderm
B) contact dentin
C) contact stellate reticulum
D) are touched by odontoblast processes
E) are touched by stratum intermedium
D) are touched by odontoblast processes

328.

Tonsils

Describe the following:


1. Pharyngeal Tonsils
2. Palatine tonsils
3. Lingual tonsils
- non-encapsulated secondary immune system
1. no lymph, sinuses, crypts
- surrounded by connective tissue and epithelium that forms deep infoldings
(ciliated pseudostratified columnar epithelium)
- adenoids can block airflow
2. crypts and lymphoid follicles, no sinuses
- covered partly by connective tissue and partly by epithelium (non-keratinized
stratified squamous)
3. lymphoid follicles, each with single crypt
- covered w// non-keratinized stratified squamous epithelium; partially
surrounded by connective tissue

329.

What is the source of NADPH?

What biological processes is it needed for?


HMP shunt

1. FA and steroid synthesis


2. glutathione reduction inside RBCs (protects from radicals and hydrogen
peroxide)

330.
The secretion of which of the following endocrine glands is NOT
essential to life?
A) Adrenal medulla
B) Anterior pituitary
C) Adrenal cortex
D) Pancreatic islets (Langerhans)
E) Parathyroids
A) adrenal medulla

331.
What is a striated duct?
- (intralobular duct) gland duct that connects intercalated duct w/ the
interlobular duct
- characterized by basal infoldings on basal surface of plasma membrane
- found in submandibular and parotid gland
- A LOT of mitochondria on basal surface
- Along with the intercalated ducts, they function to modify salivary fluid by
secreting HCO3-and K+ and reabsorbing Na+ and Cl- using the Na-K pump
and the Cl-HCO3 pump.

332.
The alternate loosening and tightening of a primary tooth that is about to
be shed results from
A) alternate resorption and apposition of cementum and bone.
B) alternate softening and hardening of alveolar bone proper.
C) labial movement of the developing permanent tooth.
D) lack of a developing permanent successor.
A

333.
A deficiency of vitamin A in a developing tooth most likely affects the
A) dentin.
B) cementum.
C) enamel.
D) pulp.
C enamel

334.
Which of the following enzymes catalyzes the formation of uric acid
from purines?
A) Xanthine oxidase
B) Urease
C) Aspartate transcarbamylase
D) Carbamyl phosphate synthetase
E) Uricase
A

335.

Long bones of the skeleton increase in length because of

A) interstitial growth in the cartilaginous epiphyseal plate


.B) resorption of primary bone by osteoclasts.
C) mitotic division of osteocytes.
D) mitotic division of osteoblasts.
E) appositional growth on the cartilaginous epiphyseal plate
E

336.
Lactose, Maltose, glucose, galactose and fructose are these kinds of
sugars because
Reducing, they contain a free anomeric carbon

337.
GAG found in synovial fluid, vitreous humor, ECM of LCT
hyaluronate, large polymers and shock absorbing

338.
GAG found in cartilage, bone and heart valves
Chondroitin sulfate, most abundant GAG

339.
GAG found in basement membrane, components of cell surfaces
heparin sulfate, contains higher acetylated glucosamine than heparin

340.
GAG found in intracellular granules of mast cells lining the arteries of
teh lungs, liver and skin
Heparin, also serves as anticoagulant, more sulfated than heparin sulfate

341.
GAG found in skin, blood vessels and heart valves
dermantan sulfate

342.
GAG found in cornea, bone, cartilage aggragated with chondroitin
sulfates
keratin sulfate, most heterogenous GAG

343.
glucose + glucose
maltose

344.
glucose + galactose
lactose

345.
glucose + fructose
sucrose

346.
This type of cell absorbs monosaccharides
enterocytes

347.
large insoluble carbohydrate thats an important plant energy source
comprised of amylose(unbranched) and amylopectin(branched)
starch

348.
Highly branched polymer of glucose that is especially abundant in the
liver
glycogen, which can be cleaved by glucan transferase

349.
common organic earth compound that is not digestible by humans and is
often referred to as dietary fiber
cellulose

350.
This is a polysaccharide of glucose produced extracellularly bacteria and
yeast.
Dextrans; notable bacteria = strep mutans

351.
This is stored intracellularly as reserve nutrients, and increases the
adhesion of bacteria to surfaces of teeth and promote the formation of dental
plaque
levans, formed from fructose and side product of dextran production

352.
structure that covers the from of the eye, bends light, and does not
change shape
cornea

353.
crystalline lens that focuses light, "fine-tunes" vision, this can become
cloudy=cataract
lens

354.
opening in the middle of the iris
pupil

355.
functions as the shutter of the eye allowing light in and contains the
color
iris

356.
thin layer of nerve tissue that senses light, made of rods and cones
retina

357.
densely packed nerve cells where focus of object or regard
fovea

358.
Constriction of the pupil of the eye
miosis

359.
prolonged abnormal dilation of the pupil induced by drug or disease
mydriasis

360.
far objects are focuses at a point in front of the retina, cornea is steeper
or eye is longer
myopia, nearsightedness

361.
treatment for myopia
concave lens

362.
light focuses behind the retina, flatter cornea or shorter eye
hyperopia, farsightedness

363.
How do you treat farsightedness?
convex lens

364.
curvature of lens is not uniform
astigmatism; treat with cylindric lenses

365.
inability of the eye to focus sharply on nearby objects because of loss of
elasticity of lents with advancing age
presbyopia, treat with bifocals

366.
Abnormal type of hemoglobin, lysine is replaced with glutamic acid
causing reduced plasticity of RBCs
Hemoglobin C

367.
Abnormal hemoglobin with four beta chains, usually associated with a
defect in three of four alpha chain genes resulting in alpha-thalassemia
hemoglobin H

368.
Abnormal hemoglobin where valine has replaced glutamic acid in beta
chain, sickle shape
hemoglobin S

369.
abnormal hemoglobin with single AA substitution favors formation
methemoglobin and is associated with methemoglobinemia
Hemoglobin M

370.
Hormones activated by foodstuffs entering the duodenum
CCK, Secretin, gastric inhibitory peptide(GIP)

371.
Reflex of the enteric nervous system?
enterogastric reflex triggered by distention of the sm intestine, adn chemical
and osmotic irritation of mucosa, inhibitory impulse

372.
most common lipid storage disease caused by a deficiency of
enzyme glucocerebrosidase. Fatty material will collect and cause the spleen and
liver to enlarge, liver malfunction, skeletal disorders, bone lesion that may
cause pain, severe neurological comp, lymph node swelling, anemia, yellow
spots in eyes. Ashkenazi Jewish ancestry
Gaucher disease

373.
autosomal recessive disorder caused by accumulation of fat and
cholesterol in liver, spleen, bone marrow, lungs, and brain. Ataxia, eye
paralysis, brain degeneration, learning problems, spasticity, feeding and
swallowing difficulties. Deficiency of sphingomyelinase and ashkenazi jewish
ancestry.
Niemann-Pick disease

374.
rare inherited disease, progressive destruction of brain and spinal cord
nerve cells. Deficiency in beta-hexosaminidase A and accumulation of GM2
gangliosides. jewish ancestry, CNS degeneration, die by 5
Tay-sachs disease

375.
autosomal recessive disorder caused by the deficiency of
galactosylceramidase
Krabbe disease

376.
alpha-galactosidase A deficiency disease, build up of fatty material in
ANS, eyes, kidneys, and cardiovascular system
Fabry disease

377.
Symptoms:
Goiter, thickened nails that lift off the nail beds, myxedema, clubbing,
exophthalmos
Grave's disease- type of hyperthyroidsim
myxedema=lumpy reddish thick skin on front of shins and sometimes the top of
feet
exopthalmos= bulging eyes

378.
autoimmune reaction to the adrenal cortex with weight loss, muscle
weakness, fatigue, Low BP, darkening of skin.
Addisons disease, adrenal glands do not produce enough cortisol and
sometimes aldosterone

379.
Disorder failure to produce ADH or in rare cases for the kidney to
respond to ADH. Increased urination and thirst and pale urine
diabetes insipidus

380.
This molecule unwinds the helix
helicases

381.
this molecule is responsible for unwinding supercoiled DNA to allow
DNA polymerase access to replicate the genetic code
topoisomerases

382.
this enzyme re-forms the supercoiled structure once the replication fork
has passed
DNA gyrase

383.
Pyrimidine bases
DNA- T and C
RNA- U and C
CUT down the pyramids

384.
Purine bases
A and G

385.
Glycolysis rate-limiting step?
PFK, phosphofructokinase, catalyzes phosphate from ATP to fructose-6phosphate

386.
Aldolytic reaction of glycolysis
Aldolase converts fructose-1,6-biphosphate to two 3-carbon metabolites

387.
Where does glycolysis occur and what are the major points?
Cytoplasm, absence of oxygen
Needs 2 ATP to start
End product is pyruvate, two molecules of NADH+ H+ are reduced, and four

molecules of ATP via substrate phosphorylation so there is a Net gain of 2


Pyruvate has two fates: aerobic respiration in mitochondria or
fermentation(anaerobic respiration)

388.
Which amino acids are not transaminated?
lysine, serine, threonine

389.
Arrange the three furcations of a maxillary first molar from closest to
farthest from the cervical line.
(a) Facial (b) Mesial (c) Distal
CBA

390.
Which of the following teeth have sharp demarcations between pulp
chambers and pulp canals?
A) Maxillary lateral incisors B) Mandibular second premolars C) Mandibular
canines D) Maxillary first premolars
D because of the mesial root concavity

391.
The contact area, a self-protective feature of the dentition, functions to:
(a) prevent food impaction;
(b) distribute occlusal forces;
(c) protect mucosal tissues;
(d) form embrasures;
(e) stabilize the dental arch.
A) (a), (d) and (e)
B) (c) and (d)
C) (e) only
D) (b) and
(d)E) (a), (b) and (c)
A

392.
Which of the following primary teeth has a distinctly prominent facial
cervical ridge that makes it uniquely different from other teeth?
A) Mandibular first molar
B) Maxillary central incisor
C) Maxillary second molar
D) Mandibular canine
E) Maxillary canine
A. mandibular 1st molar

393.
Identify the permanent anterior tooth that most frequently exhibits a
bifurcated root and identify the positions of the roots
Mandibular canine: facial and lingual

394.
When an adult with normal occlusion moves his mandible from right
lateral relation to centric occlusion, which cusp moves between facial cusps of
the maxillary right second molar?
A) Distal cusp of the mandibular first molar
B) Distofacial cusp of the mandibular second molar
C) Mesiofacial cusp of the mandibular third molar
D) Mesiofacial cusp of the mandibular second molar
E) Distofacial cusp of the mandibular first molar
B

395.

In an adult, which of the following occur normally while swallowing?

(a) Masseter muscles contract;


(b) The suprahyoid group of muscles relaxes;
(c) Teeth come into occlusal contact;
(d) The tip of the tongue touches the roof of the mouth.
A) (a), (c) and (d)B) (a), (b) a n d (c)C) (b), (c) and (d)D) All of the aboveE)
(a), (b) and (d)
A)
- palatoglossus brings tongue up and back

- suprahyoid muscles contract to raise hyoid; pharynx pulls up (epliglottis


closes airway)
- pharyngeal constrictors perform wavelike action (CN IX, X, XI)
- nasopharynx blocked off

396.
In an otherwise normal arrangement, which of the following teeth are
most often in abnormal relation and contact with adjacent teeth in the same
arch?
maxillary lateral incisors

397.

Assuming occlusion and alignment are normal, the arrow on the sketch
represents the path taken by the
A) mesiolingual cusp of a first molar.
B) facial cusp of a second premolar.
C) lingual cusp of a second premolar.
D) facial cusp of a first premolar
E) lingual cusp of a first premolar.
C

398.
Which of the following permanent teeth have mesial concavities that
require special attention when removing calculus deposits?: (a) Maxillary
central incisors; (b) Maxillary first premolars; (c) Maxillary first molars; (d)
Mandibular second premolars
B and C
- Maxillary 1st premolar has mesial concavity
- Maxillary 1st molars- only tooth with pronounced distal concavity at CEJ

399.
The facial masticatory mucosa (attached gingiva) is narrowest on which
mandibular tooth?
1st premolar

400.

The mandibular movement indicated isA) left lateral, working side.B)


protrusive.C) left lateral, non-working side.D) right lateral, working side.E)
right lateral, non-working side.
left lateral non-working side

401.
In which of the following molars is the mesial fossa most distinctly
separated from the remainder of the occlusal table by a transverse ridge?
A) Maxillary secondB) Mandibular secondC) Mandibular firstD) Maxillary
first
C

402.
In a normal occlusion, mandibular central incisors contact maxillary
incisors in which of the following movements?:
(a) Protrusive; (b) Lateral (working); (c) Lateral (non-working); (d) Lateral
protrusive
A and D

403.
A lingual pit is most common on which of the following teeth?
maxillary lateral incisor
- also a distolingual groove

404.

A) Left lateral, working sideB) ProtrusiveC) Left lateral, non-working sideD)


Right lateral, working sideE) Right lateral, non-working side
A

405.
From a proximal view, which of the following permanent teeth tends to
be positioned in the arch with its axis most nearly vertical?
maxillary canine

406.
The lingual surface of the crown of a mandibular canine is smooth and:
(a) relatively flat in the fossa area; (b) poorly developed in the marginal ridge
area; (c) poorly developed in the cingulum area.
A) (a) and (b)B) (c) only C) All of the aboveD) (a) onlyE) (b) onlyF) (b) and
(c)
C all of the above

407.
How soon after eruption of a permanent tooth is the apex usually fully
developed?
2-3 yrs

408.
The design of a restored occlusal surface is dependent upon the:
(a) contour of the articular eminence;
(b) position of the tooth in the arch;
(c) amount of lateral shift in the rotating condyle;
(d) amount of vertical overlap of anterior teeth.
all of the above

409.
The oblique ridge on a permanent maxillary first molar is reduced in
height in the center of the occlusal surface and is nearly level with the

A) triangular ridge of the mesiofacial cusp.B) marginal ridge.C) None of the


aboveD) cusp tip of the distofacial cusp.
B marginal ridge

410.

A) mesiofacial cusp of a third molar.B) distofacial cusp of a second molar.C)


distolingual cusp of a second molar.D) mesiofacial cusp of a second molar.E)
mesiolingual cusp of a second molar.
B

411.

During nonmasticatory swallowing, teeth are usually

A) in contact in intercuspal position.


B) protruded.
C) None of the above. This is a nonexistent act.
D) in a working arrangement.
A

412.
The tooth most likely to exhibit a lingual groove that extends from the
enamel onto the cemental area of the root is a permanent
maxillary lateral incisor

413.
When viewed from the mesial or the distal, the overall facial outline
from cusp tip to root apex of a mandibular canine is:
(a) made up of two arcs;
(b) made up of one continuous arc;
(c) different from the outline of a maxillary canine;
(d) very similar to the outline of a maxillary canine.
A) (b) and (c)B) (a) and (c)C) (d) onlyD) (a) and (d)E) (b) and (d)
A) B and C

- maxillary canine has a more prominent labial ridge

414.
The ideal position and height of lingual cusps of a mandibular first molar
accommodate which of the following?
A) Non-working movement
B) Centric relation
C) Working movement
D) Maximum intercuspation
E) Protrusive position
C working movement

415.
Which of the following is most commonly associated with development
of gastrointestinal cancer?
A) Pedunculated adenoma
B) Meckel's diverticulum
C) Duodenal peptic ulcer
D) Diverticulosis
E) Villous adenoma
Villous adenoma- villous adenomas are VILLANOUS

416.
Removal of the capsule from an encapsulated bacterium is likely to
result in.
A) increased susceptibility to mutation.
B) failure of the cell to gram stain.
C) loss of viability.
D) increased susceptibility to phagocytosis
.E) decreased generation time.
D

417.
The alum adjuvant employed in many vaccines acts biologically in
tissues to

A) prolong antigen release to immunocompetent cells in the region.


B) Both (1) and (2) above
C) Neither (1) nor (2) above
D) serve as a local inflammatory stimulus.
B
-decreases the need for titers and a smaller amount of antigen; increases
immune response

418.
The most reliable gaseous sterilizing agent available for dental
instruments is
A) carbon dioxide.B) hydrogen sulfide.C) ethylene oxide.D) formaldehyde.
ethylene oxide

419.
What is a hemangioma?
A benign self-involuting tumor of endothelial cells with unknown cause
most common tumor in infancy
"blood vessel tumor"

420.
Most common tumor of salivary glands?
Where?
Most common malignancy of salivary glands?
Pleomorphic (mixed) adenoma
-Parotid Gland> submandibular > Sublingual
- plasmacytoid cells are diagnostic
Mucoepidermoid Carcinoma
- mucous secreting cells and and epithelial cells
- accompanied by pain
- may show damage to CN VII

421.

Ludwig's angina

- life-threatening cellulitis infection of floor of mouth; usually in dental


infection
- angina= severe pain
- usually from infection of mandibular molars
- mixed infection- usually Streptococci always present
1) brawny/muscular induration that doesn't pit on pressure
2) rapid onset
3) Dysphagia, dyspnea, fever
- EMERGENCY when swelling blocks airway
- Tx- get rid of infection w/ antibiotics (penicillin)

422.

Amyloidosis

1. Primary
2. Secondary
3. Hereditary
proteinaceous material (amyloid) deposits and accumulates in tissues and
organs
1. unknown cause
- malignant plasma cells abnormally produce a lot of Ig's that accumulate and
deposit onto tissues
- systemic: HEART, lungs, skin, tongue, thyroid gland, intestines, liver, kidney,
blood vessels
- e.g. Multiple Myeloma (CRAB- hypercalcemia, renal disease, Anemia, Bone
pain)

2. reactive; complication of other diseases (TB, rheumatoid arthritis, etc)


- heart rarely involved
3. rare; genetic mutations

423.
Down Syndrome
- Trisomy 21 chromosomal abnormality
- mental retardation
- large protruding tongue
- small ears, large forehead, wide space eyes
- simean crease; short, broad hands
Complications
- congenital heart disease
- increased infections
- ALL (we ALL fall DOWN)
- increased Alzheimers

424.
Cri Du Chat Syndrome
- Chromosome 5 short arm deletion
1. severe mental retardation
2. microcephaly
3. catlike cry

425.
Edward's Syndrome
- Trisomy 18
1. mental retardation
2. pinched face appearance
3. micrognathia (Small jaw)
4. small head w/ low set ears
5. congenital heart disease

Das könnte Ihnen auch gefallen